NUR 2502 EXAM 2/NUR2502 EXAM 2 LATEST 2023-2024 EXAM ALL 50 QUESTIONS AND CORRECT ANSWERS/MDC3 EXAM 2 /MULTIDIMENSIONAL CARE 3 EXAM 2 |ALREADY GRADED A+|RASMUSSEN COLLEGE

Nur2502 exam 2 quizlet
Nur2502 exam 2 questions and answers
Nur2502 exam 2 questions
Nur2502 exam 2 answers
mdc 3 exam 2 rasmussen quizlet
mdc 3 exam 2 rasmussen kahoot
mdc3 exam 2 quizlet

hyperventilation, if someone is blowing off too much CO2 they become more
Alkaline, respiratory alkalosis

if a patient’s lung are not functioning very well and they are unable to remove or blow off CO2 very well, the CO2 will build up in their system becoming
acidic, respiratory acidosis

Perfusion
is adequate arterial blood flow through the peripheral tissues (peripheral perfusion) and blood that is pumped by the heart to oxygenate major body organs (central perfusion)

upper respiratory tract
nose, sinuses, pharynx, larynx

Lower respiratory tract
Lungs,Trachea, two mainstem bronchi, lobar, segmental, and subsegmental bronchi; bronchioles; alveolar ducts; alveoli

common cause of respiratory ailments
Cigarette smoke

Nursing care of a patient experiencing upper respiratory system disorders
maintaining a patent airway to allow adequate ventilation and oxygenation.
Along with a focused respiratory assessment, the nurse will utilize information obtained from the patient and family during the admission history interview. Information regarding the patient’s history of upper respiratory disorders, smoking, and environmental exposures will be utilized to determine the necessary testing and treatment

Normal Changes in Aging Adults
Alveoli function decreases
Ability to cough decreases
Lungs loose residual volume, vital capacity and gas exchange decreases.
Respiratory muscles atrophy
Vascular resistance increases, capillary flow decreases
Susceptibility to infection increases.

The turbinates
three bones that protrude into the nasal cavities from the internal portion of the nose
increase the total surface area for filtering, warming, and humidifying inspired air before it passes into the nasopharynx.

The paranasal sinuses
air-filled cavities within the bones that surround the nasal passages
Lined with ciliated membrane, the sinuses provide resonance to speech, decrease the weight of the skull, and act as shock absorbers in the event of facial trauma..

Fremitus refers to vibratory tremors that can be felt through the chest by palpation, Increased fremitus may indicate
compression or consolidation of lung tissue, as occurs in pneumonia.

Lung sounds
Bronchial
Bronchovesicular
Vesicular

Adventitious sounds
Crackles
Wheezes
Rhonchus
Pleural friction rub

Other Indicators of Respiratory Adequacy
Cyanosis, decreased capillary refill, clubbing of nails in fingers, level of consciousness, Chest Circumference, Anxiety, Dyspnea Orthopnea, General Appearance

Diagnostic Assessment of lungs
Laboratory assessment

  • RBC
  • ABG- is a blood gas and this tells us the acid base balance of the patient
  • Sputum- can tell us if microorganisms are growing in the lung – describe color, clarity, and any odor

Imaging assessment

  • x-rays-Xrays show us areas of opaque which usually indicate pneumonia/consolidation of fluid
    -CT- computed tomography. Lung nodules, areas of fluid buildup

Other noninvasive diagnostic assessments

  • Pulse oximetry-circulating O2- tells us oxygen levels in the tissues- usually fingers, toes, or earlobes
  • Capnometry and capnography-how much CO2 is leaving the lungs.
    -PFTs-Lung function- tell us how well the lungs function at moving air in and out
  • Exercise testing-Exercise tolerance

Invasive Diagnostic Assessment
-Endoscopic examinations
-Bronchoscopy- is a camera that looks at the airway passages
-Thoracentesis- can remove fluid buildup from the lung
-Lung biopsy- is used to diagnose some lung diseases or cancer

Which assessment finding for an older adult patient does the nurse ascribe to the natural aging process?
A.Tightening of the vocal cords
B.Decrease in residual volume
C.Decrease in the anteroposterior diameter
D.Decrease in respiratory muscle strength

D. As a person ages, vocal cords become slack, changing the quality and strength of the voice; the anteroposterior diameter increases; respiratory muscle strength decreases; and the residual volume increases.

The nurse knows that under normal physiologic conditions of tissue perfusion, a patient will have what percent of oxygen dissociate from the hemoglobin molecule?
A.25%
B.50%
C.75%
D.100%

ANS: B
Oxygen dissociates with the hemoglobin molecule based on the need for oxygen to perfuse tissues. Under normal conditions, 50% of hemoglobin molecules completely dissociate their oxygen molecules when blood perfuses tissues that have an oxygen tension (concentration) of 26 mm Hg. This is considered a “normal” point at which 50% of hemoglobin molecules are no longer saturated with oxygen.

Which assessment finding does the nurse interpret that is associated most closely with lung disease?
A.Cough
B.Dyspnea
C.Chest pain
D.Sputum production

ANS: A

Cough is a main sign of lung disease. Dyspnea (difficulty in breathing or breathlessness) is a subjective perception and varies among patients. A patient’s feeling of dyspnea may not be consistent with the severity of the presenting problem. Sputum production may be associated with coughing and indicate an acute or chronic lung condition. Chest pain can occur with other health problems, as well as with lung problems.

Head and Neck Cancer
Squamous cell carcinoma and slow growing
Begins with mucus that is chronically irritated, becoming tougher and thicker
Leukoplakia and erythroplakia lesions
Spreads to local lymph nodes, muscle and bone, then to liver or lungs.

Risks of head and neck cancer
•Tobacco use
•Alcohol use
•Voice abuse
•Chronic laryngitis
•Exposure to chemicals
•Poor hygiene
•Long-term gastroesophageal reflux disease
•Oral infections with human papillomavirus

Assessment: Noticing- head and neck cancer
Lumps in mouth, throat, neck
Difficulty swallowing
Color changes in mouth or tongue
Oral lesion or sore that does not heal in 2 weeks
Persistent, unilateral ear pain
Persistent/unexplained oral bleeding
Numbness of mouth, lips, or face
Change in fit of dentures
Hoarseness or change in voice quality
Persistent/recurrent sore throat
Shortness of breath
Anorexia and weight loss
Change in fit of dentures
Burning sensation when drinking citrus or hot liquids

The priority interprofessional collaborative problems for patients with head and neck cancer include
Potential for airway obstruction
Potential for aspiration
Anxiety
Decreased self esteem

Planning and Implementation: Responding Head and Neck Cancer
Radiation therapy
has a cure rate of at least 80%
hoarseness, dysphagia, skin problems, impaired taste, and dry mouth for a few weeks after radiation therapy.

Chemotherapy
Varies based on type of cancer cells: usually Cistplatin
Intensify oral cavity side effects

Cordectomy
Vocal Cord Removal

Laryngectomy
Complete or partial removal of the Larynx and surrounding area.

Laryngectomy Postoperative Care
First priorities are airway maintenance and ventilation
Wound, flap, reconstructive tissue care
Hemorrhage
Wound breakdown
Pain management
Nutrition
Speech and language rehabilitation.

Cancer of the Nose & Sinuses
Tumors rare, benign or malignant- asian Americans higher incidence of nasal cancer.
Seen with exposure to dust from wood, textiles, leather, flour, nickel, chromium mustard gas, radium
Slow onset, resembles sinusitis
Lymph enlargement often occurs on side with tumor mass
Surgical removal is treatment; may be combined with radiation (IMRT)

the same interventions listed under the section for patients with head and neck cancers

Fracture of the Nose
Displacement of bone or cartilage can cause airway obstruction or cosmetic deformity; potential source of infection
CSF may indicate skull fracture
Interventions
Closed reduction
Rhinoplasty
Nasoseptoplasty

Causes:
•Contact sports
•Fights
•Motor vehicle accidents

Postoperative Careafter Rhinoplasty
Observe for edema and bleeding
Check vital signs every 4 hours
Change drip pad as needed
Encourage patient to remain
in a semi-fowlers position.
Decrease any forcefully
coughing or straining.

•Maintaining a semi-Fowler’s position to reduce swelling.
•Application of cool compresses to reduce pain and swelling.
•Educate the patient not to cough forcefully or strain for the first few days to prevent possible bleeding.
Monitor nasal packing for increased bleeding

Facial Trauma
can involve the mandible, maxillary, orbital, and nasal bones and the side of the face. Trauma to the mandible is classified as Le Fort I, II, and III. Le Fort III can result in extensive bleeding and bruising and result in airway obstruction that impacts gas exchange.

Priority action is airway assessment

  • Manifestations
    Stridor
    Shortness of breath/dyspnea
    Anxiety/restlessness
    Hypoxia and hypercarbia
    Decreased oxygen saturation
    Cyanosis, loss of consciousness

Epistaxis
Nosebleed is a common problem
Trauma
Hypertension
Chronic Cocaine Use

Cauterization of affected capillaries may be needed; nose is packed
Posterior nasal bleeding is an emergency!
Assess for respiratory distress, tolerance of packing or tubes
Humidification, oxygen, bed rest, antibiotics, pain medications
Position patient upright and leaning forward
Pinch bridge of the nose (pressure)
Cold Compress (vasoconstriction)
Nasal Packing if necessary

•Nursing care of a patient with epistaxis includes:
•Management of bleeding by applying direct lateral pressure to the nose for 10 minutes and the application of ice or cool compresses. If bleeding does not cease, nasal packing may be applied.
•Implement standard precautions.
•Educate the patient to maintain an upright position, such as leaning forward to prevent aspiration.
•Monitor blood pressure to prevent periods of hypertension, which could increase the chance of bleeding.
•Instruct the patient not to blow his or her nose for 24 hours to prevent clot disruption.
•Avoid straining, bending over, blowing nose forcefully

Interprofessional Collaborative Care Facial Trauma
Airway assessment
Anticipate need for emergency intubation
Tracheotomy
Cricothyroidotomy
Fixed occlusion
Débridement

If a patent airway is not able to be secured, the patient may require an emergency intervention such as a tracheotomy or cricothyroidotomy. The next priorities are controlling hemorrhage, identification of the source of bleeding, and providing aggressive fluid resuscitation to maintain patient stability. Patients who present with facial trauma also require stabilization of the head and neck until the extent of the injury can be determined. Maintain cervical alignment until diagnostic tests have cleared the patient of injury. Patients who suffer from facial trauma often require surgical intervention for the injuries, nursing care of these patients includes monitoring airway, assessing for bleeding and infection, and providing patient education on oral care. Depending on the extent of the injury, the patient will likely have their jaw wired. Educate the patient and family on the use of a wire cutter for emergency situations.

obstructive sleep apnea (OSA)
cessation in breathing while sleeping.
Must occur a minimum of 5x/hour (can be hundreds/night)
lasts from 10sec – greater than 1 minute with each episode

Risk factors of obstructive sleep apnea (OSA)
Obesity
Oropharyngeal edema
Family history
Hypothyroidism
short neck with recessed chin
Enlarged tonsils, adenoids, uvula
Cigarette smoking and alcohol or sedative use

Complications of sleep Apnea
HTN
Stroke
Cognitive deficits
Weight gain
Diabetes
Pulmonary disease
Cardiovascular disease
Excessive daytime sleepiness, irritability, inability to concentrate

Diagnostic Tests for sleep apnea
STOP-Bang Sleep Apnea Questionnaire
ABG and TSH level
Sleep study: observation and measurement of the client during sleep.

T/X sleep apnea
Treatment
Lose weight if sleep apnea is caused by obesity
Refrain from alcohol or sedatives
Avoid sleeping on your back (position fixing)
Noninvasive positive-pressure ventilation (NPPV)
BiPap
CPAP
APAP
Modafinil (Attenance,Provigil)

Surgery
Adenoidectomy- to remove excess tissue
Uvulopalatopharyngoplasty (UPPP)- done for those with enlarged tissues- remodels posterior oropharynx
Tracheostomy- bypasses obstruction. Done for those with severe OSA or those who cannot tolerate the CPAP

upper airway obstruction
Airflow blockage in the nose, mouth, pharynx, or larynx

caused by:
trauma
blockages/masses
burns
foreign bodies

Medical emergency!!
Prompt action is required to prevent further patient compromise. Interventions are based on the patient’s presenting signs and symptoms and the cause of the obstruction. If the patient is conscious, perform the Heimlich maneuver. If the patient is unconscious, open the airway by repositioning the head or inserting an oral airway. Suction the patient to remove secretions and perform abdominal thrust. If the obstruction is not able to be cleared, the patient may require an emergency tracheotomy, cricothyroidotomy, or endotracheal intubation.

A 58-year-old woman who has been diagnosed with throat cancer 1 week ago comes to the clinic today to discuss surgical options with her health care provider. She is very tearful and appears sad when the nurse calls her back to the examination room.
Based on her diagnosis, which clinical manifestation will the nurse likely observe in the patient?
A.Hoarseness
B.Severe chest pain
C.Low hemoglobin level (anemia)
D.Numbness and tingling of the face

ANS: A

The patient may experience several different symptoms. The most commonly seen with throat cancer is hoarseness, as well as mouth sores or a lump in the neck. Anemia can result if surgery is performed. Severe pain in the chest can be associated with many different disorders and is not usually linked to throat cancer. Numbness and tingling of the face cannot be observed.
S/Sx of throat cancer: hoarsness, soar throat, difficulty swallowing, mouth sores, ear pain, oral bleeding

When the nurse begins taking the patient’s history, the patient asks, “Did you know that I have throat cancer and may not survive?” What is the appropriate nursing response?

A.”Are you having difficulty swallowing?”
B.”My mother had cancer, so I know how you must be feeling right now.”
C.”I am sure that your cancer can be cured if you follow your doctor’s advice.”
D.”I know you have been diagnosed with cancer. Are you concerned about what the future may hold?”
ANS: D

Although option A is part of an appropriate history, the patient’s need at the moment, represented by her statement, is psychosocial in nature. The nurse should realize that the patient may need psychosocial support. This is the only appropriate therapeutic response. The nurse cannot give her false reassurance (option C), and the nurse should never compare feelings (option B). Head and neck cancer is curable when treated early.

The provider discusses radiation therapy with the patient because her lesion is small and the cure rate is 80% or higher. The patient asks if her voice will return to normal. What is the appropriate nursing response? (Select all that apply.)

A.”At first the hoarseness may become worse.”
B.”The more you use your voice, the quicker it will improve.”
C.”Gargling with saline may help decrease the discomfort in your throat.”
D.”Your voice will improve within 4 to 6 weeks after completion of the therapy.”
E.”You should rest your voice and use alternative communication during the therapy.”
ANS: A, C, D, E

The patient should be taught not to use her voice more than necessary during and after therapy, and to work with family to determine alternative forms of communication until after the radiation therapy. Statements A, C, D, and E are appropriate responses that accurately reflect the normal course of progression after radiation therapy for throat cancer.

After the radiation therapy begins, the patient visits the clinic stating that her throat is sore, she is having difficulty swallowing, and the skin on her throat is red, tender, and peeling.

What patient teaching should the nurse provide?
ANS: For temporary relief of the patient’s sore throat and swallowing difficulty, suggest that she gargle with saline, suck on ice chips, use mouthwash, or use a throat spray with local anesthetics such as lidocaine. For her red, tender, peeling skin, have her avoid exposure to sun, heat, cold, or abrasive treatments such as shaving; wear protective clothing of soft cotton; wash gently with mild soap; and use only lotions or powders prescribed by the radiation oncologist until the area has healed.

The nurse is caring for a patient admitted for treatment of neck and throat cancer. Which intervention should the nurse perform?

A.Encourage hydration with water.
B.Feed the patient if coughing occurs.
C.Encourage the patient to sit in a chair for meals.
D.Encourage the patient to drink juice to address thirst.
ANS: C

Several interventions are necessary to reduce the risk of aspiration. Having the patient sit upright to eat is an important initial step to reduce aspiration. Other interventions include encouraging liquids that are “thick.” Avoiding thin liquids like juice, water, and fruits that produce juice are important strategies to reduce aspiration risks. Coughing may be a sign of difficulty with swallowing or aspiration and requires additional assessment.

What can happen if a pt aspirates? What are they at risk for? What are other ways we can reduce aspiration risk? (speech consult, to chair or HOB all they way up, slow eating)

The nurse is caring for a patient admitted to the ED after experiencing a fall while rock climbing. The patient has several facial fractures. Which objective assessment finding requires immediate intervention?

A.Malaligned nasal bridge
B.Blood draining from one of the nares
C.Crackling of the skin (crepitus) upon palpation
D.Clear glucose positive fluid draining from nares
ANS: D

Blood or clear fluid (cerebrospinal fluid, or CSF) may drain from one or both nares. However, the presence of glucose in the clear drainage indicates that CSF is draining, which could be caused by a skull fracture, a serious complication CSF leak- yellow ring on testing strips. A malaligned nasal bridge and crepitus may be observed when evaluating general facial fractures and would be considered an expected finding.
What would be the priority for this patient? Open airway

The nurse recognizes that a patient with sleep apnea may benefit from which intervention(s)? (Select all that apply.)

A.Weight loss
B.Nasal mask to deliver BiPAP
C.A change in sleeping position
D.Medication to increase daytime sleepiness
E.Position-fixing device that prevents tongue subluxation
ANS: A, B, C, E

All interventions listed are viable interventions that can be of benefit to patients who have sleep apnea. Patients should work with their providers of care to determine the severity of their sleep apnea and which specific interventions would be of most importance to them. Encouraging daytime sleepiness is the opposite of the effect needed for this patient.
What are the signs of sleep apea/who is at risk? Overweight, large neck size, short neckObesity
Oropharyngeal edema
Family history
Hypothyroidism
short neck with recessed chin
Enlarged tonsils, adenoids, uvula
Cigarette smoking and alcohol or sedative use
Complications: HTN
Stroke
Cognitive deficits
Weight gain
Diabetes
Pulmonary disease
Cardiovascular disease
Excessive daytime sleepiness, irritability, inability to concentrate
Treatment:Lose weight if sleep apnea is caused by obesity
Refrain from alcohol or sedatives
Avoid sleeping on your back (position fixing)
Noninvasive positive-pressure ventilation (NPPV)

With which client does the nurse anticipate complications from obstructive sleep apnea following abdominal surgery?

A. 28-year-old who is 80 lbs (36.4 kg) overweight and has a short neck
B. 48-year-old who has type 1 diabetes and chronic sinusitis
C. 58-year-old who has had gastroesophageal reflux disease for 10 years
D. 78-year-old who wears upper and lower dentures and has asthma
•A – overweight and short neck
•Age doesn’t really matter

Mr. Sherwood is a 27-year-old male who had a fractured nose and is recovering from a rhinoplasty. He has a moustache dressing in place that is dry and intact. The nurse observes that the patient is swallowing repeatedly.

What complication does the nurse anticipate? What equipment does the nurse need to assess Mr. Sherwood?

  • Posterior nasal bleeding; penlight
    •Rationale: Assessing how often the patient swallows after nasal surgery is a priority because repeated swallowing may indicate posterior nasal bleeding. A penlight is used to examine the throat for bleeding.

Mr. Sherwood is concerned because his nose keeps bleeding. He asks the nurse, “Can you tell me again what I can do to keep my nose from bleeding?”

How should the nurse respond to Mr. Sherwood’s question?

Mr. Sherwood is discharged home. The nurse talks with him and his family on how to care for Mr. Sherwood after discharge.

What are some talking points that the nurse should include in discharge teaching for Mr. Sherwood and his family?
•Answer: The nurse may suggest that the patient keep his mouth open while sneezing, not bend over, and avoid coughing and vomiting. Avoid taking aspirin and NSAIDs while the nose heals. Avoid straining during bowel movements.
•Rationale: These activities increase blood pressure causing fragile blood vessels to break and bleed. Teaching the patient to avoid these activities will prevent increase in pressure. Laxatives or stool softeners may help to ease bowel movements. Aspirin and NSAIDs increase bleeding potential.

•The nurse may include instructing the patient to stay in a semi-Fowlers’ position, to move slowly, to keep all follow-up appointments, to call his provider if fever develops, and to use a humidifier.
•Rationale: Providing discharge instructions to Mr. Sherwood and his family allows them to be involved in his care and increases compliance and health care outcomes. Correct positioning and moving slowly decrease chances of bleeding and edema to the area. Following up with the provider allows opportunity for extension of care to complete recovery and identification of problems early.

Asthma occurs in two ways:
•Inflammation
•Airway hyperresponsiveness leading to bronchoconstriction

Asthma is classified based on how well controlled the symptoms are and the patient’s response to the medications to treat the disease process. Status asthmaticus is a severe life-threatening condition that requires prompt intervention

Pathophysiology of asthma
•Intermittent and reversible airflow obstruction affecting airways only, not alveoli

Airway obstruction
•Inflammation
•Airway hyperresponsiveness

causes of asthma
exposure to allergens or irritants; stress, cold, and exercise

Interprofessional Collaborative Care for asthma
Assessment: Noticing
•Physical assessment/clinical manifestations
•Audible wheeze, increased respiratory rate
•Increased cough
•Use of accessory muscles
•”Barrel chest” from air trapping
•Long breathing cycle
•Cyanosis
•Hypoxemia

Symptoms- labored breathing, wheezing, trouble sleeping, frequent cough, feeling tired, feeling short of breath
Common triggers – pollution, dust, smoke, pet dandner, household chemicals, bacteria, viruses, mold

Assessment: Noticing for asthma
Laboratory assessment
•ABGs

Pulmonary function tests
•Forced vital capacity (FVC)
•Forced expiratory volume in first second (FEV1)
•Peak expiratory flow rate (PEFR)

Interventions: Responding to Asthma
1.Self-management education
•Personal asthma action plan
•Control and prevent flair-ups.
•Avoid Triggers
•Use of Peak-flow meter

2.Drug therapy
•Control therapy drugs (used daily)
•Reliever drugs (used to stop an attack)
•Bronchodilators
•Anti-inflammatory agents

Medications for asthma
bronchodilators
•Beta2 agonists
•Short-acting Beta agonist- Albuterol (Proventil, Ventolin)
•Long acting Beta agonist- salmeterol (Serevent)

  • Rescue medications are short-acting- Albuterol – carry with them at all times in case of an acute asthma attack – S/E of albuterol- tachycardia, headache dizziness, insomnia, nausea

Cholinergic Antagonists/anticholinergic/long-acting muscarinic antagonists
•Tiotropium (Spiriva), ipratropium (Atrovent)

Anti-Inflammatory Agents
•corticosteroid- fluticasone (Flovent), budesonide (Pulmicort), prednisone
•Leukotriene modifiers- montelukast (Singular), zafirlukast (Accolate)

•Corticosteroids—Disrupt production pathways of inflammatory mediators. The main purpose is to prevent an asthma attack caused by inflammation or allergies (controller drug) – Teach patient to use good mouth care and to check mouth daily for lesions or drainage because these drugs reduce local immunity and increase the risk for local infections, especially Candida albicans (yeast).

•Leukotriene Modifier—Blocks the leukotriene receptor, preventing the inflammatory mediator from stimulating inflammation. The purpose is to prevent an asthma attack triggered by inflammation or allergens.

Asthma treatment continued
Other treatments for Asthma:
•Exercise and activity to promote gas exchange
•Oxygen therapy

Patient Education
•Avoid triggers
•Stop or avoid smoking
•Teach which inhaler is rescue, which is not rescue
•Use rescue inhaler 30 mins before exercise
•Know how to use inhalers (chart 30-7 and 30-8 in book)

status asthmaticus
•Severe, life-threatening, acute episode of airway obstruction
•Intensifies once it begins, often does not respond to common therapy
•Patient can develop pneumothorax and cardiac/respiratory arrest

T/X
IV fluids
potent systemic bronchodilator
steroids
epinephrine
oxygen

chronic obstructive pulmonary disease (COPD)
•Characterized by bronchospasm and dyspnea
•Tissue damage not reversible; increases in severity, eventually leads to respiratory failure
•Cigarette smoking is the greatest risk factor
•Alpha1-antitrypsin deficiency
4th leading cause of morbidity in U.S

The inflammatory response calls cell mediators to the airway. These cell mediators injure cells, over time can permanently damage them.
The repair process after the injury results in scar tissue, non-elastic tissue to form in the lung tissue/airway

2 TYPES:
•Emphysema- alveolar membrane breakdown
•Chronic bronchitis-inflammation and excessive mucus in the bronchiole tubules

Causes: smoking, pollution, genetics

•S/S – chronic cough, mucus, fatigue, SOB, chest pain and discomfort, dyspnea

Emphysema
Pink Puffer- thin and frail-looking

occurs due to high levels of proteases in the lung, which damage the alveoli and cause air trapping in the alveoli.
Emphysema is classified as panlobular, centrilobular, or paraseptal.

  • Elastin in the lungs broken down by a high level of proteases damaging alveoli
    •Airways collapse/narrow in time
    •Flattens/weakens diaphragm
    •Hyperinflation of lung and air trapping
  • severe dyspnea
    -quiet chest
  • xray- infiltration with flattened diaphrams.

Chronic Bronchitis
Blue bloater

•Inflammation of bronchi & bronchioles (airway only)
•Inflammation, vasodilation, mucosal edema, congestion, bronchospasm
•Mucus plugs and infection narrow the airway

  • overweight and cyanotic
  • elevated hemoglobin
  • Ronchi and wheezing
  • peripheral edema

Complications of COPD
Gas Exchange:
•Hypoxia
•Acidosis
Decreased Oxygen and Increased Carbon Dioxide

•Respiratory Infection
Increased Mucus Production, Inflammation and Bronchospasms

•Heart Failure- Cor pulmonale- Right-sided heart failure caused by pulmonary disease.
•Dysrhythmia
Respiratory Failure

COPD assessment
•Assessment
•Three primary symptoms
•Cough, sputum production, and dyspnea on exertion
•Lung sounds: Wheezes or crackles (rarely)
•Prolonged expiratory phase
•Distant heart sounds
•Orthopneic position, uses chest and abd muscles to help breathe
•Weight loss- due to dyspnea with eating
•Progressive airflow obstruction leads hypoxia
•Clubbing- bulbous enlargement of distal fingers and nails- associated with chronic cyanosis
•Psychosocial

COPD Tests, Education & Nursing interventions
•Laboratory Assessments & Diagnostic Tests:
•Pulmonary function/ spirometry testing
•Chest X-ray- reveals heart and lung size
•CT of the Chest- shows hyperinflation and/or bullae
•ECG- dysrhythmias
•ABG- blood gases

Education
•Quitting smoking is the most effective way to slow the progression of COPD
•Energy conservation
•Small frequent meals (high calorie/protein)- stay hydrated!
•Rest before meals if dyspneic

Interventions
Other nursing care strategies include performing chest physiotherapy to facilitate the clearance of secretions, encouraging the patient to cough and take deep breathes as well as the use of the incentive spirometer to facilitate airway clearance. Patients may also benefit from noninvasive ventilation to assist with ventilation and oxygenation.
•Patients with emphysema often have limited levels of activity due to their disorder; it is important to implement strategies to prevent deep vein thrombosis such as anti-embolism stockings or sequential compression stockings. Patients who require surgical management will require nursing interventions, including preparing the patient for the procedure and post-operative management of the patient.

COPD treatment and meds
COPD Exacerbation treatment
•Low flow oxygen- DO NOT LIMIT 1-2 L, not above 4L
•Rest (limit activities to focus on breathing and oxygenation)
•Increase fluids (to help thin secretions making it easier to expel them)
•Continuous pulse oximetry monitoring

•Meds- Nebulizer treatments with normal saline or a mucolytic agent such as acetylcysteine (Mucosil, Mucomys & same as asthma
•And possible anti-tussives like guaifenesin and dextromethorphan (Mucinex DM) also raises the cough threshold.
Sat goal is 88-92% they may not get higher than that!

cystic fibrosis
•An inherited, recessive, chronic, progressive, and frequently fatal disease of the body’s exocrine mucus-producing glands that primarily affects the respiratory, digestive, and intestinal systems and pancreas.

Each parent must pass the recessive gene for the child to get CF.

Leads to lung infections, poor digestion, poor food absorption, and male sterility

•Genetic disease affecting many organs, lethally impairing pulmonary function
•Blocked chloride transport, producing thick mucus with low water content
•Mucus plugs up glands, causing atrophy and organ dysfunction

Signs and symptoms of Cystic fibrosis
Non-pulmonary symptoms
•Abdominal distention
•GERD, rectal prolapse, foul-smelling stools, steatorrhea
•Malnourishment, vitamin deficiencies

Pulmonary symptoms
•Respiratory infections
•Chest congestion and sputum production
•Decreased pulmonary function
•Limited exercise tolerance

Nonsurgical Management of Cystic Fibrosis
•Nutritional management
•Preventive/maintenance therapy
•Exacerbation therapy

Nutrition
•Teach the importance of pancreatic enzyme replacement, adequate oral fluid intake, and diet to promote removal of secretions
•Monitor stools for bulky, foul-smelling stool (indicates malabsorption)
•Give supplemental iron
•Monitor daily weight
•Daily chest CPT

Implementation of daily chest physiotherapy with postural drainage will assist in drainage of secretions and improve oxygenation. Infection is common in patients with this disorder; early identification and treatment are vital. Patients may also benefit from noninvasive positive-pressure ventilation to improve ventilation. Other nursing interventions include pre-operative and post-operative care for patients undergoing a lung transplant

Surgical Management CF
•Lung transplantation
•Does not cure
•Extends life by 1 to 15 years
•Transplant rejection rate is high
•Patient at continued risk for lethal pulmonary infections

Pulmonary Arterial Hypertension (PAH)
idiopathic pulmonary hypertension
•Occurs in absence of other lung disorders; cause unknown
•Blood vessel constriction with increasing vascular resistance in the lung
•Heart fails (cor pulmonale)
•Without treatment, death within 2 years

•Nursing care of the patient with pulmonary hypertension is dependent on the severity classification. Nursing care interventions include administration of prescribed medications (such as calcium channel blockers, endothelin-receptor agonist, natural and synthetic prostacyclin agents, and guanylate cyclase stimulators), administration of oxygen as prescribed, assisting with placement of a pulmonary artery catheter to monitor pressures, and preparation of the patient for surgical intervention.

Idiopathic pulmonary fibrosis
•Common, restrictive lung disease
•Progressive disease, with few periods of remission
•Loss of cellular regulation

Lung injury> Inflammation >Fibrosis> Scarring

  • Corticosteroids, other immunosuppressants mainstays of therapy

•The patient usually is an older adult with a history of cigarette smoking, chronic exposure to inhalation irritants, or exposure to the drugs amiodarone (Cordarone) or ambrisentan (Letairis, Volibris). Most patients have progressive disease with few remission periods. Even with proper treatment, most patients usually survive less than 5 years after diagnosis
Causes of Pulmonary Fibrosis – The cause of IPF is unknown and the course of the disease is not predictable

•Pulmonary Fibrosis
•Nursing interventions for a patient with pulmonary fibrosis include administration of the medications and oxygen as prescribed and preparation of the patient for a lung transplant. Patient and family education include identification of community resources, monitoring for respiratory infections and identification of when to seek evaluation, and oxygen safety. Other strategies for patients and families facing end stages of the disease include providing information to the patient and family regarding hospice care.

Lung cancer
Lung cancer develops due to a loss of cellular regulation; the most common site of lung cancer is the epithelium of the bronchial tree. Lung cancers are classified as small cell lung cancer (SCLS) and non-small cell lung cancer (NSCLC).
Lung cancers impede gas exchange, which disrupts the acid-base balance. If left untreated, lung cancer can metastasize to other organs and tissues. Early recognition and aggressive treatment are essential to improve the patient’s prognosis. Primary prevention is the key to reducing the incidence of lung cancer.

Lung Cancer Warning Signs
•Hoarseness

  • Change in respiratory pattern
  • Persistent cough or change in cough
  • Blood-streaked sputum
  • Rust-colored or purulent sputum
  • Frank hemoptysis
  • Chest pain or chest pressure
  • Shoulder, arm, or chest wall pain
  • Recurring episodes of pleural effusion, pneumonia, or bronchitis
  • Dyspnea
  • Fever associated with one or two other signs
  • Wheezing
  • Weight loss
  • Clubbing of the fingers

Nonsurgical Management lung cancer
•Chemotherapy treatment of choice for lung cancers, especially small cell lung cancer (SCLC) Side effects that occur with chemotherapy for lung cancer include chemotherapy-induced nausea and vomiting (CINV), alopecia (hair loss), open sores on mucous membranes (mucositis), immunosuppression with neutropenia, anemia, thrombocytopenia (decreased numbers of platelets), and peripheral neuropathy.

•Targeted therapy- •common in the treatment of non-small cell lung cancer (NSCLC). These agents take advantage of one or more differences in cancer cell growth or metabolism that is either not present or only slightly present in normal cells. Agents used as targeted therapies work to disrupt cancer cell division

•Radiation therapy may be performed before surgery to shrink the tumor and make resection easier

•Photodynamic therapy may be used to remove small bronchial tumors using targeted laser lights

Surgical Management lung cancer
•Lobectomy
•Pneumonectomy
•Segmentectomy
•Wedge resection

•Post Op- most pt have very high levels of intense pain afer lung surgery – may have PCA
•Assess 02 sats continuously and perform lung assessment frequently- usually have suppl O2 via nasal cannula
•Coughing- splint any coughing with a pillow
•Use incentive spirometer q2 hour and get up to edge of bed or chair as soon as able

chest tube placement
•To remove fluid or air from the pleural space.
•Usually inserted into 5th intercostal space creating a communication between the outside atmosphere and the pleural space. . . . Allows air/fluid to move out.
•As the air/fluid is removed, the pleural space is decompressed to restore negative pressure.
•The chest tube is hooked to a canister measuring fluid output, it may also be hooked up to suction.
•The canister has a water seal, which prevents air from flowing back into the pleural space.

Chest Tube Drainage System
nursing management of a chest tube
you are going to check them for patency, make sure they are draining, make sure there is still pressure going to the chest tube so that the negative pressure environment in the lung can be maintained- you want to note the amount of drainage per hour (usually you make a small mark with perm marker on the drain) color/clarity of the drainiage- usually it is sereous to serosang but occasionally it is white (A chylothorax (ky-low-thor-ax) is the buildup of chyle (kile), a milky white fluid, in the space around the lungs. … Fluid can collect between the pleura layers (called the pleural space). A chylothorax happens when the lymphatic system starts leaking chyle fluid into the pleural space.)

This fluid is measured hourly during the first 24 hours. The fluid in chamber one must never fill to the point that it comes into contact with any tubes! If the tubing from the patient enters the fluid, drainage stops and can lead to a tension pneumothorax.

A chest tube falling out is an emergency. Immediately apply pressure to chest tube insertion site and apply sterile gauze or place a sterile vasaline/occlusive dressing gauze and dry dressing over insertion site and ensure tight seal. Apply dressing when patient exhales. If patient goes into respiratory distress, call a code

Nursing Care for Chest Tubes-The Patient
•Ensure that the dressing on the chest around the tube is tight and intact. Depending on agency policy and the surgeon’s preference, reinforce or change loose dressings.
•Assess for difficulty breathing.
•Assess breathing effectiveness by pulse oximetry.
•Listen to breath sounds for each lung.
•Check alignment of trachea.
•Check tube insertion site for condition of the skin. Palpate area for puffiness or crackling that may indicate subcutaneous emphysema.
•Observe site for signs of infection (redness, purulent drainage) or excessive bleeding.
•Check to see if tube “eyelets” are visible.
•Assess for pain and its location and intensity and administer drugs for pain as prescribed.
•Assist patient to deep breathe, cough, perform maximal sustained inhalations, and use incentive spirometry.
•Reposition the patient who reports a “burning” pain in the chest.

Nursing Care for Chest Tubes-The Drainage System
Keep drainage system lower than the level of the patient’s chest.

•Do not “strip” the chest tube.

Keep the chest tube as straight as possible from the bed to the suction unit, avoiding kinks and dependent loops. Extra tubing can be loosely coiled on the bed.

  • Ensure that the chest tube is securely taped to the connector and that the connector is taped to the tubing going into the collection chamber.
  • Assess bubbling in the water-seal chamber; should be gentle bubbling on patient’s exhalation, forceful cough, position changes.
  • Assess for “tidaling” (rise and fall of water in chamber three with breathing).
  • Check water level in the water-seal chamber and keep at the level recommended by the manufacturer.
  • Check water level in the suction control chamber and keep at the level prescribed by the surgeon (unless dry suction system is used).

Clamp the chest tube only for brief periods to change the drainage system or when checking for air leaks.

  • Check and document amount, color, and characteristics of fluid in the collection chamber as often as needed according to the patient’s condition and agency policy.
  • Empty collection chamber or change the system before the drainage makes contact with the bottom of the tube.
  • When a sample of drainage is needed for culture or
    other laboratory test, obtain it from the chest tube; after cleaning chest tube, use a 20-gauge (or smaller) needle and draw up specimen into a syringe.

Nursing Care for Chest Tubes-Emergencies
•Tracheal deviation
•Sudden onset or increased intensity of dyspnea
•Oxygen saturation less than 90%
•Drainage greater than 70 mL/hr
•Visible eyelets on chest tube
•Chest tube falls out of the patient’s chest (first, cover the area with dry, sterile gauze)
•Chest tube disconnects from the drainage system (first, put end of tube in a container of sterile water and keep below the level of the patient’s chest)
•Drainage in tube stops (in the first 24 hours)

Interventions for Palliation
•Oxygen therapy
•Drug therapy
•Radiation therapy
•Thoracentesis and pleurodesis
•Dyspnea management
•Pain management
•Hospice care

Based on the patient’s diagnosis, which clinical manifestations would the nurse expect to see when assessing this patient? (Select all that apply.)

A.Bradycardia
B.Shortness of breath
C.Use of accessory muscles
D.Sitting in a forward posture
E.Barrel chest appearance
ANS: B, C, D, E

The patient with COPD often has a barrel chest appearance, is short of breath, and may use accessory muscles when breathing. These patients tend to move slowly and are slightly stooped. Usually they sit with a forward-bending posture. With severe dyspnea, they exhibit activity intolerance and activities such as bathing and grooming are avoided.

While the Rapid Response Team is at the bedside, the patient’s healthcare provider arrives. The provider writes several orders.

Which order is most important for the nurse to implement immediately?

A.Transfer to ICU
B.Increase O2 to 3 L per nasal cannula
C.ABGs 30 minutes after oxygen is increased
D.Methylprednisolone sodium succinate (Solu-Medrol) 40 mg IVP
ANS: B

All of the provider’s orders are very important, but based on the patient’s severe shortness of breath, the first thing that should be done is to increase her oxygen. Once her oxygen is increased, the nurse should note the time and remember to call for stat ABGs in 30 minutes. The patient should then be transferred to the ICU as soon as possible. Once the patient arrives in the ICU, they can administer the one-time dose of Solu-Medrol.

A patient with COPD presents for a routine follow up. The patient smokes 1 PPD. Which statement by the patient causes the nurse to suspect an increase in dyspnea?

A.”I bought a new pillow so I could prop myself up at night to sleep.”
B.”I have a productive cough in the morning.”
C.”I have gained weight since I was here last.”
D.”The patient is well groomed and is sitting in a tripod position.”
ANS: A

Patients with COPD, who smoke, may have a productive morning cough. Weight loss often occurs when dyspnea is increased due to the increased metabolic demand. A tripod or orthopneic position is common with COPD and when combined with a disheveled appearance may indicate an increase in dyspnea. Buying a new pillow indicates that the patient must sleep propped up because breathing is worse while lying down. They may not recognize the increased dyspnea and they try to compensate by using multiple pillows in order to rest.

The nurse is assessing a patient with a chest tube following a pneumonectomy. Which assessment finding requires intervention?

A.Bandage around the posterior tube is loose.
B.2 cm of water is in the second chest tube chamber.
C.The water in the water seal chamber rises and falls with inhalation/exhalation.
D.Bubbling present in the water seal chamber when the patient coughs.
ANS: A

After lung surgery, two tubes, anterior and posterior, are used. Dressings around the wound should not be loose. The wounds should be covered with airtight dressings.

A patient with a history of asthma is having shortness of breath. The nurse discovers that the peak flowmeter indicates a peak expiratory flow (PEF) reading that is in the red zone. What is the priority nursing action?

A.Administer the rescue drugs.
B.Take the patient’s vital signs.
C.Notify the patient’s prescriber.
D.Repeat the PEF reading to verify the results.
ANS: A

A PEF reading in the red zone indicates a range that is 50% below the patient’s personal best PEF reading and indicates serious respiratory obstruction. The patient needs to receive rescue drugs immediately, and then the prescriber should be notified. Repeating the PEF reading and taking vital signs are also important, but doing so first delays the administration of the rescue drugs.

Let’s talk about meds! Ok so when someone has an asthma attack what is first? Albeuterol rescue inhaler (SABA short acting bronchodilator), THEN what would be give? Then we could give them their inhaled steroid (pulmicort or fluticasone) OR if we are in the hospital they sometimes can get an IV corticosteroid like methyprednisalone as well to reduce inflacmation

Seasonal Influenza
•Highly contagious acute viral respiratory infection: Strains A, B and C
•Flu season ranges from October – May with peak in February
•Severe headache, muscle ache, fever, chills, fatigue, weakness, anorexia
•Patient is contagious 24hours before to up to 5 days after onset of symptoms.
•Vaccination is advisable
•Age, Chronic illness, Pregnancy, Immunocompromised at high risk
•Antiviral agents may be effective if started within 24 to 48 hours of symptoms

Pandemic Influenza
•Mostly prevalent among animals and birds; virus can mutate, becoming infectious to humans
These symptoms can progress to pneumonia. Preventative measures, including yearly influenza vaccination, can prevent pandemic issues.
•Examples
•H1N1 (swine flu)
•H5N1 (bird flu)
•Strict isolation precautions
•Antiviral drugs
•Oseltamivir (Tamiflu), zanamivir (Relenza)

Pneumonia
•Acute or Chronic infection of one or both lungs caused by bacteria, virus, chemical irritant, fungus, mycoplasms, toxic gasses, aspiration of water, food, fluid (saliva), vomit…
•Increased risk population (table 31-1)
•Less than 2 yrs
•Older than 65 yrs
•Immunocompromised patient
•Pneumocystis carinii pneumonia (PCP) is a type of fungal infection that mostly affects pts with HIV. Can be deadly as this affects immunocompromised patients.
•Types of Pneumonia
•Community Acquired pneumonia (CAP)-
•Hospital Acquired pneumonia (HAP)
•Ventilator-associated pneumonia (VAP)
•Health Care-associated

Pneumonia Assessment
•Assessment
•Fever, chills/rigors, sweats
•Cough with or without sputum
•Pleuritic chest pain/weakness
•Dyspnea
•Malaise
•Fatigue
•Headaches
•Anorexia
•RR>20, O2 may be needed, HR increased
•Lung sounds: crackles auscultated, wheezing

•Older adults- hypotensive with orthostatic hypotension d/t vasodilation and dehydration – potentially confusion/falls

Pneumonia tests, interventions and treatment
Diagnostic Tests
•Chest x-ray
•CBC
•Sputum sample
•Blood urea nitrogen (dehydration)

Interventions
•Oxygen therapy
•Incentive spirometry
•Fluids

Medication
•Antibiotic therapy
•Pneumonia vaccine
Bronchodilators, especially beta2 agonists (see Chart 30-6), are prescribed when bronchospasm is present. They can be given by nebulizer or metered-dose inhaler. Inhaled or IV steroids are used with acute pneumonia when airway swelling is present. Expectorants such as guaifenesin (Mucinex) may be used. Antibiotics are used if bacterial infection is suspected culprit

Appropriate antibiotics are prescribed. A chest tube(s) to closed-chest drainage is used to promote lung expansion and drainage

Pneumonia education
•Frequent oral hygiene
•Good hand washing to reduce spread of infection
•If pt is ventilated, needs oral hygiene every 1-2hrs with chlorhexidine
•Change oxygen tubing with contamination
•Importance of vaccination-Pneumovax or Prevnar 13
•Pt receiving antibiotics that is having rigors, increased fever, or increased cough with sputum may indicate the medication is ineffective.
•Increase oral fluids to 2L/day to help thin sputum and clear secretions
•Incentive Spirometer every 1h WA, or TCDB

•The Joint Commission National Patient Safety Goals [NPSGs] recommend that nurses especially encourage adults older than 65 years and those with a chronic health problem to receive immunization against pneumonia. There are two pneumonia vaccines: pneumococcal polysaccharide vaccine (PPSV 23), known as Pneumovax; and pneumococcal conjugate vaccine (PCV-13), known as Prevnar 13

Pulmonary empyema
•is a collection of pus in the pleural space most commonly caused by pulmonary infection. When empyema is present, GAS EXCHANGE can be impaired by both reduced lung diffusion and reduced effective ventilation. Empyema fluid is thick, opaque, exudative, and foul smelling.

Tuberculosis
Airborne precautions

•Chronic infection of the lung caused by mycobacterium results in tubercles

•Tubercles- nodules or swelling of lymphocytes and epithelioid cells that form lesions in lung tissue

Assessment: (symptoms are often vague)
•2+ weeks of cough, fever or weight loss
•Weight loss over 3lbs/wk is considered significant
•Night sweats, weakness, and chills may be present with hemoptysis when the infection has progressed

Diagnostic Tests
•PPD or Mantoux skin testing. Results are read 48-72h after injection
•Chest Xray
•Sputum culture
•NAAT

Tuberculosis treatment and education
Medication
•Rifampin (RIF)
•Isoniazid (INH)
•Pyrazinamide (PZA)
•Ethambutol (EMB)
•These medications are given in different combinations that are tailored to the patient’s individual infection
•All cause hepatotoxicity!- check liver enzymes

Nursing Practice
•Importance of medication regimen adherence
•Direct observation Therapy- PHN observes patients in their homes with their medication regimens
•Respiratory isolation
•Education
•How to minimize the spread of TB
•Potential of resistant TB if medication not followed
•Encourage to eat a healthy diet (anorexia is common)

Rhinosinusitis
•Inflammation of nasal mucosa
•”Hay fever” or “allergies”

Manifestations
•Headache
•Nasal irritation and congestion
•Sneezing and rhinorrhea

Interventions
•Antihistamines, leukotriene inhibitors, mast cell stabilizers, decongestants, antipyretics, antibiotics
•Supportive therapy
•Complementary and alternative therapy—vitamin C, zinc

Peritonsillar Abscess (PTA)
•Complication of acute tonsillitis
•Manifestations
•Pus causing one-sided swelling with deviation of the uvula
•Trismus and difficulty breathing
•Bad breath, swollen lymph nodes
•Treatment
•Percutaneous needle aspiration of abscess
•Antibiotics

•. Diagnosis is usually made based on the patient’s symptoms, but needle aspiration and culture of pus collected is the preferred test.

•trouble swallowing, trouble breathing, difficulty speaking, drooling, or any other signs of potential airway obstruction would be ER visit

Pertussis
•Highly contagious, bacterial respiratory infection.
•First phase (catarrhal)
•Symptoms resemble the common cold
•Second phase (paroxysmal)
•Severe coughing, coughing spasms
•Thick exudate in the small airways
•Third phase (convalescent)
•Recovery (can last for months)

•During the catarrhal phase, which lasts for one to two weeks, the patient appears to have a common cold. The nose and airways become filled with mucus. During the paroxysmal phase, the patient experiences severe coughing episodes that may result in vomiting. During the convalescent phase, the patient is recovering; this stage can last several months.

Coccidioidomycosis
•Coccidioidomycosis is a fungal infection caused by the Coccidioides organism
•Common in the desert southwest regions of the United States, Mexico, and Central and South America
•Also known as “Valley Fever”
•Respiratory symptoms, headache, muscle aches, chest pain, fever
•Treated with antifungal medication

•The incubation period is one to four weeks. Inhaled spores stimulate an inflammatory process in the lungs resulting in an infection. These spores are present in the soil and, when disturbed, are released into the air.
can lead to development of an actual pulmonary infection within 1 to 4 weeks after exposure

risk factor
•Agricultural workers

resemble other respiratory infections with fever, cough, headache, muscle aches, chest pain, and night sweats. Bone and joint pain indicates more severe infection. Often the disorder is misdiagnosed and mistreated as influenza or pneumonia. Neither antibacterial drugs nor antiviral drugs are effective therapy. The disease can become widespread and cause symptoms of hemoptysis, meningitis, and involvement of the skin, adrenal glands, liver, and spleen. It also can become chronic and debilitating

The nurse understands that which of the following is the most common symptom of pneumonia in the older adult patient?

A.Fever
B.Cough
C.Confusion
D.Weakness
ANS: C

The older adult with pneumonia often has weakness, fatigue, lethargy, confusion, and poor appetite. Fever and cough may be absent, but hypoxemia is usually present. The most common manifestation of pneumonia in the older adult patient is confusion from hypoxia rather than fever or cough.

What do we need to teach our older adults about pneumonia:
Complete antibiotics as prescribed, rest , drink fluids, and minimize contact with crowds

A patient presents to the primary healthcare provider’s office with fever, ear pressure, sore throat, nasal congestion, and poor response to decongestants. What condition does the nurse suspect?

A.Pneumonia
B.Peritonsillar abscess
C.Tuberculosis exposure
D.Bacterial rhinosinusitis
ANS: D

The presented symptoms indicate bacterial rhinosinutsitis. Symptoms in bacterial infection include purulent nasal drainage with postnasal drip, sore throat, fever, erythema, swelling, fatigue, dental pain, and ear pressure. A lack of response to decongestants can also be indicative of a bacterial infection.

A nursing student is teaching a 72-year-old patient about the importance of the pneumonia vaccination. Which teaching requires intervention by the nurse? (Select all that apply.)

A.”You will only need one vaccine called Pneumovax.”
B.”You will need two vaccines to prevent pneumonia.”
C.”If you have had the Prevnar vaccine, then you will not need the Pneumovax vaccine.”
D.”Since you are over 64 years old, only the flu vaccine is suggested.”
E.”You will receive the Prevnar vaccine about a year after the Pneumovax vaccine.”
ANS: A, C, D,

There are two pneumonia vaccines: pneumococcal polysaccharide vaccine (PPSV 23), known as Pneumovax; and pneumococcal conjugate vaccine (PCV-13), known as Prevnar 13. The CDC recommends that adults older than 65 years be vaccinated with both, first with Prevnar 13 followed by Pneumovax about 6 to 12 months later. Adults who have already received the Pneumovax should have Prevnar 13 about a year or more later. These recommendations also apply to adults between 19 and 64 years of age who have specific risk factors such as chronic illnesses (CDC, 2015j).

never confirm/deny anything
“tell me more about it”

Therapeutic Communication Qs…

airborne
– N95
– negative pressure room

What precautions are we using for tuberculosis?

antimicrobials
– isoniazid,
– rifampin
– pyrazinamide
– ethambutol

*they will have to take these “long-term” (6-9 mo.)*

Medications to treat tuberculosis

– no ETOH
– medication compliance – stick with the doses, continue all the way through
– f/u with doctor appointments

Patient Education: Long-term antibiotics

– sputum culture

*PPD test is NOT for diagnosing, it does NOT mean active exposure*

How do you diagnose tuberculosis?

hepatotoxic

– no ETOH
– no drugs
– eat a diet with quality protein; iron; vitamins A, B, C, and E; and abundant fresh produce
– no other use of hepatotoxic medications

What do antimicrobials do to the body? And what patient education goes along with that?

asthma

__________ is a chronic intermittent airway obstruction caused by inflammation of the airway tissues that results in *bronchoconstriction*

▪ Intermittent and *reversible airflow obstruction affecting airways only, not alveoli​​*

complete airway obstruction

intubation/tracheotomy….keep that airway patent – priority!

I think this is about status asthmaticus:

A patient comes in with asthma…you listen to their lungs and they have lung sounds…then you lose lung sounds…what does that mean? What do we do next?

▪ pollution
▪ dust
▪ smoke
▪ fire places
▪ pet dander
▪ household chemicals
▪ bacteria
▪ viruses
▪ mold
▪ exercise
▪ weather changes from warm to cold
▪ drugs: aspirin, NSAIDS, beta blockers

What can exacerbate asthma?

“A good education question…avoid triggers!”

bronchodilators
– beta2 agonists (SABAs) – albuterol, levalbuterol, terbutaline


corticosteroids
– fluticasone
– budesonide
– mometasone

leukotriene modifiers
– montelukast/Singular

Medications to treat asthma

5 min apart
albuterol (bronchodilator) first
rinse mouth after fluticasone (why? to avoid candida/thrush)

How do you take asthma medications? Ex: Spiriva and albuterol

improve gas exchange

Overall goal for Emphysema/COPD patients

Yes! Permanent damage to alveoli. Not reversible.

Are emphysema/COPD permanent?

CO2 builds up because the alveoli can’t properly open for the CO2 to be expelled.

What happens to the patient’s CO2 levels in COPD?

tripod pose
barrel chest
accessory muscle use
clubbing
SOB
cyanosis
dyspnea

S/S: Emphysema/COPD

anxious – we have to decrease their anxiety; can’t give them sedatives because that will decrease their respirations. So we to use relaxation techniques, etc.

progressive relaxation, hypnosis therapy, and biofeedback (Ch 30)

How will emphysema/COPD patients be feeling? + patient education

▪ Respiratory infections
▪ Wheezing
▪ Dyspnea
▪ Tachypnea
▪ Sputum production – thick mucus
▪ Chest congestion
▪ Barrel chest
▪ Distended abdomen
▪ Crackles
▪ Clubbing of the fingers and toes
▪ Dry, nonproductive cough
▪ Foul-smelling pale stool with high-fat content
▪ Hematemesis
▪ Poor growth
▪ Limited exercise tolerance
▪ Decreased pulmonary function

S/S: Cystic Fibrosis

sweat test (chloride)

How do you diagnose cystic fibrosis?

*osteoporosis – teach patients to have Ca intake*
decreased immunity
buffalo hump
thin skin
weight gain
hyperglycemia
peptic ulcer disease
increased potential for infection
adrenal insufficiency

Side effects of long-term steroid use

▪ Nutrition management
▪ Positive expiratory pressure
▪ Chest physiotherapy
▪ Medications
▪ Oxygen Therapy
▪ Fluids – why? thins secretions…give as long as they don’t have CHF
▪ Lung Transplant – extends life by 1 to 15 years

Treatment: Cystic Fibrosis

Pulmonary fibrosis

_____________ is a common restrictive lung disease that is prevalent in the older adult. The onset is generally slow and results from loss of cellular regulation, which leads to prolonged inflammation of the lung tissue, causing scarring and fibrosis. *poor outcome…<5 yrs*

▪ lungs become scarred, stiff, and thick, and the progressive damage isn’t reversible.

Airway!

Nursing care: Manage potential airway obstructions due to the presence of edema or a tumor.

What’s our biggest concern with head and neck cancer?

▪ Weight loss
▪ Dysphagia
▪ Pain
▪ Lump in the mouth, throat, or neck
▪ Oral lesions that do not heal in 2 weeks
▪ Persistent, unexplained oral bleeding
▪ Numbness of the mouth, lips, or face
▪ Changes in the fit of dentures
▪ Persistent unilateral ear pain
▪ Hoarseness or change in voice quality
▪ Persistent or recurrent sore throat
▪ SOB
▪ oral leukoplakia
▪ Color changes in mouth or tongue
▪ Burning sensation when drinking citrus or hot liquids

S/S: Head and Neck Cancer

Radiation or surgery (laryngectomy), pain management with medications

How do you treat head and neck cancer?

skincare
don’t erase the marks
changes in taste
voice SHOULD go back to normal if there’s no surgery
Voice will improve 4-6 weeks post therapy & Rest voice during treatment and use alternative communication.

Patient education for radiation

▪ Manage potential airway obstructions due to the presence of edema or a tumor.
▪ Aspiration precautions
▪ Ventilation and oxygenation.
▪ Monitor arterial blood gases
▪ Assess respiratory rate and depth
▪ Monitor pulse oximetry
▪ Monitor for chemotherapy side effects: N/V

If the patient has had radiation
▪ Consulting speech therapy to assist the patient with communication.
▪ Manage sore throat by implementing pain management strategies (gargling with saline, sucking on ice, and use of mouthwash and throat sprays).
▪ Instruct the patient to refrain from being in the sun and avoid shaving.
▪ Teach the patient to wear protective clothing and gently clean skin with mild soap daily.

Post-operative management includes:
▪ Maintaining a patent airway by suctioning the patient.
▪ Monitor patient for complications of surgery such as hemorrhage, airway obstruction, wound breakdown, and possible infection.
▪ Implement nutritional support to meet the patient’s needs. A feeding tube is generally inserted at the time of surgery to provide nutritional support since oral intake is not possible.
▪ Consult speech and language pathologist to assist patient with communication needs.

Patient and family teaching for diagnosis includes:
▪ Education on performing tracheostomy care to include return demonstration.
▪ Education on the administration of feeding to include return demonstration.
▪ Education on signs and symptoms of infection (fever, chills, redness, drainage at sites) and when to follow up with the provider.

Nursing Care: Head and Neck Cancer

communication
diet – PEG tube

Nursing Care: Patient has a tracheostomy tube

Educate the patient not to cough forcefully or strain for the first few days to prevent possible bleeding.

Post-Rhinoplasty patient education

Image: Educate the patient not to cough forcefully or strain for the first few days to prevent possible bleeding.

sleep apnea

__________ is a condition in which the patient experiences cyclical patterns of breathing disruption for periods of 10 seconds that occurs at least five times in an hour due to upper airway obstruction.

▪ These periods of apnea decrease gas exchange, which can lead to an acid-base imbalance.
▪ Patients who suffer from this condition also suffer from excessive daytime sleepiness, inability to concentrate, and irritability.
▪ The most common cause of the obstruction is the soft palate or tongue.

Image: sleep apnea

▪ Obesity
▪ Large uvula
▪ Short neck
▪ Smoking
▪ Enlarged tonsils or adenoids
▪ Oropharyngeal edema

Who is at risk for sleep apnea?

▪ Noninvasive positive-pressure ventilation (NPPV)
▪ Bi-level positive airway pressure (BiPAP)
▪ Autotitrating positive airway pressure (APAP)
▪ Nasal continuous positive airway pressure (CPAP)
▪ Adenoidectomy
▪ Uvulectomy
▪ Uvulopalatopharyngoplasty (UPP)
▪ Tracheostomy

Treatment for sleep apnea

hypoxia

– heart disease
– CVA
– stroke

If you have sleep apnea and it’s not treated properly what could happen? (“complication”)

pneumococcal polysaccharide vaccine (PPSV 23), known as Pneumovax; and pneumococcal conjugate vaccine (PCV-13), known as Prevnar 13.

What 2 vaccinations do you need for pneumonia?

oxygen therapy
incentive spirometer
bronchodilators
steroids
expectorants
antibiotics
sepsis prevention
fluids

Treatment for pneumonia

yes, if it’s “viral” pneumonia. Tell patient to stay away from others. Take full course of antibiotics.

Is pneumonia contagious?

can’t delegate what you EAT

Evaluate
Assess
Teach

“a couple questions about delegation”

Epistaxis

bleeding from the nose

pressure
pinch nose
lean forward

▪ Capillary cauterization
▪ Silver nitrate
▪ Electrocautery
▪ Nasal packing
▪ Epistaxis catheters

▪ Manage bleeding by applying direct lateral pressure to the nose for 10 minutes and application of ice or cool compresses.
▪ Nasal packing
▪ Implement standard precautions.
▪ Educate the patient to maintain an upright position, such as leaning forward to prevent aspiration.
▪ Monitor blood pressure to prevent periods of hypertension, which could increase the chance of bleeding.
▪ Instruct the patient not to blow his or her nose for 24 hours to prevent clot disruption.

How do you treat epistaxis?

dilate vessels…keeps them patent…allows for proper perfusion and no clot formation

Medications (such as calcium channel blockers, endothelin-receptor agonist, natural and synthetic prostacyclin agents, and guanylate cyclase stimulators)

What’s the overall goal for a patient with pulmonary hypertension?

▪ Dyspnea
▪ Fever
▪ Use of accessory muscles to breathe
▪ Change in respiratory pattern
▪ Clubbing
▪ Weight loss
▪ Wheezing
▪ Persistent cough or change in cough
▪ Chest pain, chest tightness
▪ Shoulder, arm, or chest wall pain
▪ Pleural friction rub
▪ Recurring pleural effusion, pneumonia, bronchitis
▪ Enlarged lymph nodes
▪ Hemoptysis (blood-tinged sputum)
▪ Fatigue
▪ Hoarseness​

Does NOT present like a respiratory infection…it’s …chronic….and hemoptysis is involved.

S/S: Lung Cancer

another question about lung cancer she thought was “too easy”…probably about smoking cessation

sleep study

How do you diagnose sleep apnea?

A. “You will only need one vaccine called Pneumovax.”
C. “If you have had the Prevnar vaccine, then you will not need the Pneumovax vaccine.”
D. “Since you are over 64 years old, only the flu vaccine is suggested.”

QfromtextbookA nursing student is teaching a 72-year-old patient about the importance of the pneumonia vaccination. Which teaching requires intervention by the nurse? (Select all that apply.)

A. “You will only need one vaccine called Pneumovax.”
B. “You will need two vaccines to prevent pneumonia.”
C. “If you have had the Prevnar vaccine, then you will not need the Pneumovax vaccine.”
D. “Since you are over 64 years old, only the flu vaccine is suggested.”
E. “You will receive the Prevnar vaccine about a year after the Pneumovax vaccine.”

gas exchange

What is a priority when assessing a person with facial trauma?

stridor, SOB, dyspnea

S/S: Airway Obstruction

edema, asymmetry, pain, leakage of spinal fluid through the ears or nose (can be clear to pink-tinged)

S/S: Facial Trauma

▪ Bruising
▪ Pain
▪ Nasal deviation/malalignment
▪ Crepitus
▪ Blood or clear fluid draining from the nose
▪ Impaired breathing

*check for CSF*

S/S: Nasal Fractures

▪ Trauma
▪ Hypertension
▪ Leukemia
▪ Inflammation
▪ Tumor
▪ Decreased humidity
▪ Nose blowing/picking
▪ Chronic cocaine use
▪ Nasal procedure

Causes: Epistaxis

Blue Bloaters

AIRWAYS ONLY NOT alveoli.

▪ Productive cough
▪ Dyspnea
▪ Tachypnea
▪ Pursed-lip breathing
▪ Cyanosis
▪ Use of accessory muscles
▪ Pedal edema
▪ Weight gain
▪ Jugular vein distention
▪ Stridor
▪ Fever
▪ Rhonchi
▪ Wheezing

S/S: Bronchitis

▪ cigarette smoking
▪ Alpha1-antitrypsin deficiency

Causes: COPD

Hypoxia​
Acidosis​ -> decreased O2 and increased CO2
Respiratory Infection​ -> increased mucus production, inflammation, bronchospasms
Heart Failure​ -> cor pulmonale
Dysrhythmia
Respiratory Failure​​

Complications of COPD

TRUE

T/F: Cystic Fibrosis – chlorine utilization deficiency can impact multiple organs.

A nursing student is teaching a client about their new diagnosis of pulmonary fibrosis. The student would include which of the following in their teaching?
A. A sputum culture may show the presence of mycobacterium
B. This is incurable, autosomal recessive genetic disease that affects many organs.
C. Inflammation of the mucous membranes in the airways can trigger an attack.
D. Most clients have progressive disease with a life expectancy of less than5 years.

C. Inflammation of the mucous membranes in the airways can trigger an attack.

2. A nurse is providing discharge instructions to a client recently diagnosed with Tb. Which statement by the client indicates correct understanding of the teaching? SATA
A. “I will follow up with my healthcare provider regularly.”
B. “My family does not require testing”
C. “I need to strictly adhere to my medication schedule.”
D. ” I will avoid alcoholic beverages while on this treatment plan”.
E. ” I will visit the clinic every week for injections of medication”

A. “I will follow up with my healthcare provider regularly.”
B. “My family does not require testing”
C. “I need to strictly adhere to my medication schedule.”
D. ” I will avoid alcoholic beverages while on this treatment plan”.

3. The nurse is teaching the client about post-rhinoplasty care. Which statement by the client indicates an understanding of the instructions?
A. ” I should remain supine if possible.”
B. ” I should take over-the counter-nonsteroidal anti-inflammatory drug ( NSAIDs).”
C. ” I will have nasal packing and mustache dressing.”
D. ” I will be able to breathe only from my nose”

C. ” I will have nasal packing and mustache dressing.”

4. Which statement from a client with seasonal influenza requires additional teaching?
A. ” I’m contagious only when symptoms are present.
B. “I can reduce my risk by implementing good hand hygiene.”
C. I should receive a new influenza vaccine every year”
D. “I can be diagnosed on presentation of symptoms”

A. ” I’m contagious only when symptoms are present.

A nurse is providing teaching to a client recently diagnoses with sleep apnea. Which of the
following statements by the client indicates an understanding of the teaching.
A. Sleep apnea only has an impact on my mental concentration.”
B. ” I should contact the provider if my oxygen level is below 90%.
C. ” I should begin treatment only if my snoring impacts my partner.”
D. ” I should contact the provider for a prescription for sleep medication.”

B. ” I should contact the provider if my oxygen level is below 90%.

6. A nurse is preparing to administer dextromethorphan 30mg PO now. The amount available is dextromethorphan oral liquid 7.5 mg/5ml. How many ml should the nurse administer per dose? ( Record answer as a whole number)

20

7. A nurse is caring for a client who has emphysema. Which of the following findings should the nurse expect to assess in this client? SATA.
A. Weight gain
B. Wheezing
C. Tachypnea
D. Barrel chest
E. Distended jugular vein

B. Wheezing
C. Tachypnea
D. Barrel chest
E. Distended jugular vein

8. A client with suspected TB is admitted to the hospital. A long with a private room, which of the following is appropriate related to isolation procedures?
A. Respiratory isolation and contact isolation for sputum only
B. Respiratory isolation with surgical masks until diagnosis is confirmed
C. No respiratory isolation necessary until diagnosis is confirmed.
D. Negative airflow room with a specially fitted respirator

D. Negative airflow room with a specially fitted respirator

9. A nurse admits a client from the emergency department with new onset of dyspnea and productive cough with suspected pneumonia. The client has an oxygen saturation of 96% on 2L of O2 via nasal canula, and crackles in bilateral lung bases. Oral temperature 98.9 F, heart rate 103, respiratory rate 18. The provider enters the following orders, which will the nurse perform First?
A. Collect blood sample for complete blood count
B. Administer PO antipyretic for temperature over 101 degrees 101 degrees Fahrenheit
C. Administer broad spectrum antibiotic
D. Collect sputum sample for culture

d. Collect sputum sample for culture

10. An 84-year-old client is diagnosed with rhinosinusitis. The nurse questions which medication that she sees on the client’s PRN medication list?
a. Analgesic
b. Nasal spray
c. Antihistamine
d. Antipyretic

c. Antihistamine

11. A nurse caring for a 60-year-old male client recently diagnosed with neck cancer. Which of the following assessment findings is not consistent with this diagnosis?
A. Aphonia
B. Difficulty swallowing
C. Nausea
D. Weight gain

A. Aphonia

12. The nurse is assessing a client who reports being struck in the face and head several times. During this assessment, the nurse observes pink-tinged drainage from the client’s nares. What nursing.. data? (Select One)
A. Test the drainage with a regent to check the pH
B. Place a drop of drainage on filter paper and look for a yellow ring
C. Ask the client to describe the appearance of the face before the injury
D. Have the client gently blow their nose and observe for bloody mucus

B. Place a drop of drainage on filter paper and look for a yellow ring

13. The nurse knows which of the following tests is needed to confirm a tuberculosis diagnosis?
A. Mantoux skin test
B. Sputum culture
C. Complete blood count
D. Chest X-RAY

B. Sputum culture

14. A homeless client is being discharged from a long-term therapy floor for tuberculosis (TB). What referral by the nurse is most appropriate?
A. Outpatient public health visiting nurses for direct observation.
B. Physical therapy for muscle strengthening to prevent home falls.
C. Department of health for community infection control isolation
D. Occupational therapy for employment placement and housing

A. Outpatient public health visiting nurses for direct observation.

15. The nurse knows which of the following is the purpose of montelukast for a client with asthma?
A. Constricts the smooth muscles of the airway and bronchioles.
B. Acts as a rapid bronchodilator in severe asthmatic episodes.
C. Blocks leukotriene receptors to decrease inflammation.
D. Reduces the histamine effect of the triggering agents.

C. Blocks leukotriene receptors to decrease inflammation.

16. The nurse is providing discharge instruction for a client diagnosed with pneumonia. Which information is the nurse sure to include?
A. Complete antibiotics as prescribed, rest, drink fluids, and minimize contact with crowds.
B. Take all antibiotics as ordered, resumed at an all activity as before hospitalization.
C. No restrictions regarding activities, diet, and rest because the client is fully recovered when discharge.
D. Continue antibiotics only also no further sign of pneumonia is present avoid exposing immunosuppressant individuals.

A. Complete antibiotics as prescribed, rest, drink fluids, and minimize contact with crowds.

17. A Patient with a recent diagnosis of sinus cancer states that he wants another course of antibiotics because he believes he has another sinus infection. What is the best nurse response?
A. ” Why are you doubting your doctor’s diagnosis?”
B. ” Tell me more about your understanding of sinus cancer symptoms”
C. ” Let me bring you a brochure about sinus cancer”
D. I will tell the physician to order an antibiotic”

B. ” Tell me more about your understanding of sinus cancer symptoms”

18. The nurse is assessing a client admitted with status asthmaticus. Initially , the nurse heard wheezes in the lungs, but now the lung sounds are inaudible. What is the priority intervention?
A. Administration of long- acting bronchodilator.
B. Measures to reduce anxiety
C. Education to prevent future exacerbations
D. Activation of rapid response team to secure an airway

D. Activation of rapid response team to secure an airway

19. A client was recently diagnosed with laryngeal cancer. When the nurse begins taking the client’s history , the client asks,” did you know that I have a throat cancer and may not survive?

D. Tell me more about your concerns.

20. The nurse is caring for a client 1 day after receiving radiation therapy for neck cancer. Which finding would the nurse expect after radiation therapy?
A. Expressive aphasia
B. Excessive saliva
C. Mucus secretion
D. Voice hoarseness

D. Voice hoarseness

21. Which intervention promotes comfort in dyspnea management for a client with lung cancer?
A. Provide supplemental oxygen via nasal cannula or mask
B. Place the client in a supine position with a pillow under the knees and legs.
C. Encourage exercise and independent ambulation around the room.
D. Administer morphine only when the client request it (double check please)

A. Provide supplemental oxygen via nasal cannula or mask

22. A patient presented to the emergency room with difficulty breathing. Upon examination, the client has pus behind the tonsil and swelling on the right side of her neck. She is diagnosed with a peritonsillar abscess. Which of the following is a treatment priority for the patient?
A. Maintain a patient airway
B. oxygen therapy
C. analgesics
D. antibiotics

A. Maintain a patient airway

23. A nurse is caring for several older client in the hospital that the nurse identifies as being at high risk for healthcare-associated pneumonia. To reduce this risk, what activities should the nurse delegate to the unlicensed assistive personnel (UAP)?
A. Provide oral care every 4 hours.
B. Encourage between-meal snacks
C. Report any new onset of cough
D. Monitor temperature every 4 hour

A. Provide oral care every 4 hours.

24. A client with chronic bronchitis often shows signs of hypoxia. Which of the following is the priority to monitor for in this client?
A. oxygen saturation level. Large amounts of thick mucus
B. Barrel chest
C. nutritional status
D. clubbing of fingers

A. oxygen saturation level. Large amounts of thick mucus

25. In planning care for a client with chronic obstructive pulmonary disease (COPD), the nurse acknowledges what statement is true regarding nutritional needs?
A. COPD can Increase metabolism, and the client should consume supplements additional calories and protein.
B. COPD has no effect on calories and protein needs, meal tolerance, appetite, and weight.
C. A client with COPD should decrease intake of calories and protein as dyspnea causes activity intolerance.
D. COPD can cause an anabolic state, which creates conditions for building strengths and body mass.

A. COPD can Increase metabolism, and the client should consume supplements additional calories and protein.

26. A nurse is providing education to a client recently diagnosed with pulmonary hypertension. What is the goal of drug therapy for this client?
A. Increase the pulmonary vascular pressure to slow cor pulmonale.
B. Increase the client’s systemic blood pressure with vasoconstriction.
C. Reduce the pulmonary pressure to slow cor pulmonale.
D. Decrease the client’s pain and make the client comfortable.

C. Reduce the pulmonary pressure to slow cor pulmonale.

27. A nurse is caring for a client who has been diagnosed with chronic obstructive pulmonary disease. Which of the following would be a treatment priority for the client?
A. Improve gas exchange.
B. Blood pressure control.
C. Prevention of infection.
D. Increase activity level.

A. Improve gas exchange.

28. The nurse is caring for a client who was recently diagnosed with asthma and is providing education on triggers of asthma. Which of the following can potentially trigger the disease process? (SATA)
A. Cigarette smoking
B. Animal dander (pets)
c. Pollution.
d. Exercise
e. Dust.

A. Cigarette smoking
B. Animal dander (pets)
c. Pollution.
d. Exercise
e. Dust.

29. A 47-year-old male client presented to the emergency room with complaints of nasal and facial pain and bloody discharge. He states the symptoms started approximately three months ago and have gotten progressively worse. He states that it feels like his nose is blocked up all the time. Based on these symptoms, which of the following diagnostic tests would the nurse expect the provider to order?
A. Liver function test
B. Complete blood count
C. Tumor mapping
D. Computer tomography (CT) scan of the face

D. Computer tomography (CT) scan of the face

30. A client who has chronic obstructive pulmonary disease (COPD) and asthma is receiving oxygen at 2 liters per minute. A family member tells a nurse. ” My mother did not look good, so I turned her oxygen up to 7 liters”. Which of these nursing actions is best?
A. Notify the healthcare provider immediately about the family member.
B. Thank the family member and continue to observe the client on this oxygen level.
C. Decrease the oxygen to 2 liters per minute and assess the client.
D. Elevate the head of the bed to make the client more comfortable.

C. Decrease the oxygen to 2 liters per minute and assess the client.

32. A nurse is caring for a client with cystic fibrosis. Which of the following are assessment findings for a client with this disorder? (Select all that apply.)
A. Thick sticky mucus.
B. Steatorrhea.
C. Decrease forced vital capacity (FVC)
D. Recurrent respiratory infections.
E Gastroesophageal reflux disease (GERD)

A. Thick sticky mucus.
B. Steatorrhea.
C. Decrease forced vital capacity (FVC)
E Gastroesophageal reflux disease (GERD)

33. A client arrives in the emergency department with epistaxis. What is the nurse’s priority intervention?
A. Position the client upright with the head forward.
B. Monitor the color and the amount of blood.
C. Apply an ice pack to the nose.
D. Place the nasal packing.

A. Position the client upright with the head forward.

34. A client has been taking isoniazid for 3 weeks. What information gathered by the public health nurse needs to be reported to the healthcare provider immediately?
A. Client is drinking 4-6 alcoholic beverages per day.
B. Client was recently started on varenicline to quit smoking.
C. Client has been taking isoniazid daily as prescribed.
D. Client smokes 1.5 packs cigarette per day

A. Client is drinking 4-6 alcoholic beverages per day.

36. The nurse is caring for a 60-year-old female client who presented to the emergency room status post motor vehicle accident. The client was an unrestrained passenger who hit the windshield and has multiple facial lacerations and dyspnea. Which is a priority nursing intervention for this client?
A. Insert the intravenous catheter.
B. Evaluate the pulse and blood pressure
C. Assess and maintain the airways.
D. Assess the client’s breathing pattern

C. Assess and maintain the airways.

37. Anxiety is common among clients who are diagnosed with chronic obstructive pulmonary disease. Which of the following interventions can assist in reducing a client’s anxiety? (SATA.)
A. Starting a vigorous exercise routine.
B. Plan out periods of rest throughout the day.
C. Professional counselling.
D. Written plan for dealing with anxiety
E. Relaxation techniques

B. Plan out periods of rest throughout the day.
C. Professional counselling.
D. Written plan for dealing with anxiety
E. Relaxation techniques

38. A client presents with signs and symptoms that are often associated with lung cancer. Which clinical manifestations does the nurse expect to observe in this client? (SATA.)
A. Hypothermia
B. Hoarseness
C. Peripheral edema
D. Frank hemoptysis
E. Chest tightness

B. Hoarseness
D. Frank hemoptysis
E. Chest tightness

39. A nurse is teaching a 78-year-old client about the importance of the pneumonia vaccination. Which statement by the client indicates an understanding of the teaching?
A. “I ‘ve already had pneumonia, so I only need one vaccination.”
B. “I only need pneumonia vaccination upon admission to a nursing home.”
C. “I need two different vaccinations to prevent pneumonia.”
D. “Only the flu vaccination is recommended at my age.”

C. “I need two different vaccinations to prevent pneumonia.”

40. The nurse is caring for a postoperative client returning to the unit after surgical removal of cancer of the head. Which action(s) should the nurse take initially? (SATA)
A. Ensure adequate gas exchange.
B. Ambulation of the client postoperatively
C. Assess the client’s hemodynamics status
D. Monitor for airway maintenance
E. Educate the client on anesthesia effects

A. Ensure adequate gas exchange.
C. Assess the client’s hemodynamics status
D. Monitor for airway maintenance
E. Educate the client on anesthesia effects

41. A client has positive Mantoux skin test result. What explanation does the nurse give to the client?
A. “There is active disease, and you need immediate treatment.”
B. “A repeat skin test is necessary because the test could give a false-positive result”
C. “There is active disease, but you are not infectious to others”
D. “You have been infected but this does not mean active disease is present.”

D. “You have been infected but this does not mean active disease is present.”

42. A nurse is preparing to administer 250 mg of ceftriaxone IM stat. Available is ceftriaxone 1g/5 ml. how many ml should the nurse administer per dose? (Record the answer to the nearest hundredth, or two decimal places. Use a leading zero if it applies. Do not use a trailing zero. Answer numerically only, do not label)

1.3

43. The nurse is caring for a client who was recently diagnosed with cystic fibrosis. Which of the following is a treatment option for this disorder?
A. pain management
B. weight reduction
C. Chest physiotherapy
D. Tracheostomy

C. Chest physiotherapy

44. Which of the following is a major diagnostic test for cystic fibrosis?
A. Sweat chloride test
B. Chest computed tomography test
C. Arterial blood gas
D. Chest x-ray

A. Sweat chloride test

45. A Nurse is caring for client with end stage emphysema. Which of the following would be an expected finding?
A. pH 7.50
B. CO2 50mm Hg
C. CO2 30 mm Hg
D. HCO3 26 mEq/L

B. CO2 50mm Hg

46. Which of the following is a common problem associated with cystic fibrosis in adults?
A. Hypertension
B. Asthma
C. Obesity
D. Osteoporosis.

D. Osteoporosis.

47. The nurse is providing education to a client who is prescribed a long-acting beta-agonist medication. Which statement by the client indicates the client understands the teaching?
A. “I will take this medication when I start to experience an asthma attack.”
B. “I will take this medication every morning to help prevent an acute attack.”
C. “I will only take this medication when I am admitted to the hospital.”
D. “I will carry this medication with me at all times in case I need it”

B. “I will take this medication every morning to help prevent an acute attack.”

48. The nurse teaches a client with asthma to monitor for which problem while exercising?
A. Wheezing from bronchospasm.
B. Swelling in the feet and ankle
C. Muscle fatigue
D. Increased peak expiratory flow rates

a. Wheezing from bronchospasm.

49. The nurse is performing medication teaching for a client with chronic airflow limitation. What is the correct sequence for administering inhaled medications?
A. Bronchodilators should be taken 5-10 minutes after the steroid
B. Bronchodilators and steroids are two different lasses of drugs, so the sequence irrelevant
C. Bronchodilator should be taken at least 5 minutes before other inhaled drugs.
D. Bronchodilator should be taken immediately after the steroid

C. Bronchodilator should be taken at least 5 minutes before other inhaled drugs.

50. The change of shift report has just been completed on the medical surgical unit. Which client will the oncoming nurse plan to assess first?
A. client with chronic obstructive pulmonary disease (COPD) who is ready to discharge but is unable to afford prescribed medication.
B. Client with cystic fibrosis (CF) who has an elevated temperature and a newly increased respiratory rate of 38 breaths/min.
C. Hospice client with end-stage pulmonary fibrosis and an oxygen saturation level of 89%
D. Client with lung cancer who needs an intravenous antibiotic administered before going to surgery

B. Client with cystic fibrosis (CF) who has an elevated temperature and a newly increased respiratory rate of 38 breaths/min.

50. Nurse caring for a client recently diagnosed with asthma. Which is not related?

Obesity

Nurse caring for client underwent laryngectomy. Appropriate post operative care?

Pain management, alternative means of communication, diet modification, stress reduction

A client with suspected TB is admitted to hospital. Isolation precautions?

Negative airflow room with specialty fitted respirator

Which statement by client shows understanding of radiation for neck cancer?

My voice initially be hoarse but should improve over time

Which of the following is a problem with cystic fibrosis in adults?

Osteoporosis

The nurse knows which of the following is the purpose of a fluticasone inhaler?

Reduces obstruction of airways by decreases inflammation

The nurse is teaching a client about post rhinoplasty care

I should try and avoid coughing, sneezing, and blocking my nose

hyperventilation, if someone is blowing off too much CO2 they become more

Alkaline, respiratory alkalosis

if a patient’s lung are not functioning very well and they are unable to remove or blow off CO2 very well, the CO2 will build up in their system becoming

acidic, respiratory acidosis

Perfusion

is adequate arterial blood flow through the peripheral tissues (peripheral perfusion) and blood that is pumped by the heart to oxygenate major body organs (central perfusion)

upper respiratory tract

nose, sinuses, pharynx, larynx

Lower respiratory tract

Lungs,Trachea, two mainstem bronchi, lobar, segmental, and subsegmental bronchi; bronchioles; alveolar ducts; alveoli

common cause of respiratory ailments

Cigarette smoke

Nursing care of a patient experiencing upper respiratory system disorders

maintaining a patent airway to allow adequate ventilation and oxygenation.
Along with a focused respiratory assessment, the nurse will utilize information obtained from the patient and family during the admission history interview. Information regarding the patient’s history of upper respiratory disorders, smoking, and environmental exposures will be utilized to determine the necessary testing and treatment

Normal Changes in Aging Adults

Alveoli function decreases
Ability to cough decreases
Lungs loose residual volume, vital capacity and gas exchange decreases.
Respiratory muscles atrophy
Vascular resistance increases, capillary flow decreases
Susceptibility to infection increases.

The turbinates

three bones that protrude into the nasal cavities from the internal portion of the nose
increase the total surface area for filtering, warming, and humidifying inspired air before it passes into the nasopharynx.

The paranasal sinuses

air-filled cavities within the bones that surround the nasal passages
Lined with ciliated membrane, the sinuses provide resonance to speech, decrease the weight of the skull, and act as shock absorbers in the event of facial trauma..

Fremitus refers to vibratory tremors that can be felt through the chest by palpation, Increased fremitus may indicate

compression or consolidation of lung tissue, as occurs in pneumonia.

Lung sounds

Bronchial
Bronchovesicular
Vesicular

Adventitious sounds

Crackles
Wheezes
Rhonchus
Pleural friction rub

Other Indicators of Respiratory Adequacy

Cyanosis, decreased capillary refill, clubbing of nails in fingers, level of consciousness, Chest Circumference, Anxiety, Dyspnea Orthopnea, General Appearance

Diagnostic Assessment of lungs

Laboratory assessment
– RBC
– ABG- is a blood gas and this tells us the acid base balance of the patient
– Sputum- can tell us if microorganisms are growing in the lung – describe color, clarity, and any odor

Imaging assessment
– x-rays-Xrays show us areas of opaque which usually indicate pneumonia/consolidation of fluid
-CT- computed tomography. Lung nodules, areas of fluid buildup

Other noninvasive diagnostic assessments

– Pulse oximetry-circulating O2- tells us oxygen levels in the tissues- usually fingers, toes, or earlobes

– Capnometry and capnography-how much CO2 is leaving the lungs.
-PFTs-Lung function- tell us how well the lungs function at moving air in and out
– Exercise testing-Exercise tolerance

Invasive Diagnostic Assessment

-Endoscopic examinations
-Bronchoscopy- is a camera that looks at the airway passages
-Thoracentesis- can remove fluid buildup from the lung
-Lung biopsy- is used to diagnose some lung diseases or cancer

Which assessment finding for an older adult patient does the nurse ascribe to the natural aging process?

A.Tightening of the vocal cords
B.Decrease in residual volume
C.Decrease in the anteroposterior diameter
D.Decrease in respiratory muscle strength

D. As a person ages, vocal cords become slack, changing the quality and strength of the voice; the anteroposterior diameter increases; respiratory muscle strength decreases; and the residual volume increases.

The nurse knows that under normal physiologic conditions of tissue perfusion, a patient will have what percent of oxygen dissociate from the hemoglobin molecule?

A.25%
B.50%
C.75%
D.100%

ANS: B
Oxygen dissociates with the hemoglobin molecule based on the need for oxygen to perfuse tissues. Under normal conditions, 50% of hemoglobin molecules completely dissociate their oxygen molecules when blood perfuses tissues that have an oxygen tension (concentration) of 26 mm Hg. This is considered a “normal” point at which 50% of hemoglobin molecules are no longer saturated with oxygen.

Which assessment finding does the nurse interpret that is associated most closely with lung disease?

A.Cough
B.Dyspnea
C.Chest pain
D.Sputum production

ANS: A

Cough is a main sign of lung disease. Dyspnea (difficulty in breathing or breathlessness) is a subjective perception and varies among patients. A patient’s feeling of dyspnea may not be consistent with the severity of the presenting problem. Sputum production may be associated with coughing and indicate an acute or chronic lung condition. Chest pain can occur with other health problems, as well as with lung problems.

Head and Neck Cancer

Squamous cell carcinoma and slow growing
Begins with mucus that is chronically irritated, becoming tougher and thicker
Leukoplakia and erythroplakia lesions
Spreads to local lymph nodes, muscle and bone, then to liver or lungs.

Risks of head and neck cancer

•Tobacco use
•Alcohol use
•Voice abuse
•Chronic laryngitis
•Exposure to chemicals
•Poor hygiene
•Long-term gastroesophageal reflux disease
•Oral infections with human papillomavirus

Assessment: Noticing- head and neck cancer

Lumps in mouth, throat, neck
Difficulty swallowing
Color changes in mouth or tongue
Oral lesion or sore that does not heal in 2 weeks
Persistent, unilateral ear pain
Persistent/unexplained oral bleeding
Numbness of mouth, lips, or face
Change in fit of dentures
Hoarseness or change in voice quality
Persistent/recurrent sore throat
Shortness of breath
Anorexia and weight loss
Change in fit of dentures
Burning sensation when drinking citrus or hot liquids

The priority interprofessional collaborative problems for patients with head and neck cancer include

Potential for airway obstruction
Potential for aspiration
Anxiety
Decreased self esteem

Planning and Implementation: Responding Head and Neck Cancer

Radiation therapy
has a cure rate of at least 80%
hoarseness, dysphagia, skin problems, impaired taste, and dry mouth for a few weeks after radiation therapy.

Chemotherapy
Varies based on type of cancer cells: usually Cistplatin
Intensify oral cavity side effects

Cordectomy
Vocal Cord Removal

Laryngectomy
Complete or partial removal of the Larynx and surrounding area.

Laryngectomy Postoperative Care

First priorities are airway maintenance and ventilation
Wound, flap, reconstructive tissue care
Hemorrhage
Wound breakdown
Pain management
Nutrition
Speech and language rehabilitation.

Cancer of the Nose & Sinuses

Tumors rare, benign or malignant- asian Americans higher incidence of nasal cancer.
Seen with exposure to dust from wood, textiles, leather, flour, nickel, chromium mustard gas, radium
Slow onset, resembles sinusitis
Lymph enlargement often occurs on side with tumor mass
Surgical removal is treatment; may be combined with radiation (IMRT)

the same interventions listed under the section for patients with head and neck cancers

Fracture of the Nose

Displacement of bone or cartilage can cause airway obstruction or cosmetic deformity; potential source of infection
CSF may indicate skull fracture
Interventions
Closed reduction
Rhinoplasty
Nasoseptoplasty

Causes:
•Contact sports
•Fights
•Motor vehicle accidents

Postoperative Careafter Rhinoplasty

Observe for edema and bleeding
Check vital signs every 4 hours
Change drip pad as needed
Encourage patient to remain
in a semi-fowlers position.
Decrease any forcefully
coughing or straining.

•Maintaining a semi-Fowler’s position to reduce swelling.
•Application of cool compresses to reduce pain and swelling.
•Educate the patient not to cough forcefully or strain for the first few days to prevent possible bleeding.
Monitor nasal packing for increased bleeding

Facial Trauma

can involve the mandible, maxillary, orbital, and nasal bones and the side of the face. Trauma to the mandible is classified as Le Fort I, II, and III. Le Fort III can result in extensive bleeding and bruising and result in airway obstruction that impacts gas exchange.

Priority action is airway assessment

– Manifestations
Stridor
Shortness of breath/dyspnea
Anxiety/restlessness
Hypoxia and hypercarbia
Decreased oxygen saturation
Cyanosis, loss of consciousness

Epistaxis

Nosebleed is a common problem
Trauma
Hypertension
Chronic Cocaine Use

Cauterization of affected capillaries may be needed; nose is packed
Posterior nasal bleeding is an emergency!
Assess for respiratory distress, tolerance of packing or tubes
Humidification, oxygen, bed rest, antibiotics, pain medications
Position patient upright and leaning forward
Pinch bridge of the nose (pressure)
Cold Compress (vasoconstriction)
Nasal Packing if necessary


•Nursing care of a patient with epistaxis includes:
•Management of bleeding by applying direct lateral pressure to the nose for 10 minutes and the application of ice or cool compresses. If bleeding does not cease, nasal packing may be applied.
•Implement standard precautions.
•Educate the patient to maintain an upright position, such as leaning forward to prevent aspiration.
•Monitor blood pressure to prevent periods of hypertension, which could increase the chance of bleeding.
•Instruct the patient not to blow his or her nose for 24 hours to prevent clot disruption.
•Avoid straining, bending over, blowing nose forcefully

Interprofessional Collaborative Care Facial Trauma

Airway assessment
Anticipate need for emergency intubation
Tracheotomy
Cricothyroidotomy
Fixed occlusion
Débridement

If a patent airway is not able to be secured, the patient may require an emergency intervention such as a tracheotomy or cricothyroidotomy. The next priorities are controlling hemorrhage, identification of the source of bleeding, and providing aggressive fluid resuscitation to maintain patient stability. Patients who present with facial trauma also require stabilization of the head and neck until the extent of the injury can be determined. Maintain cervical alignment until diagnostic tests have cleared the patient of injury. Patients who suffer from facial trauma often require surgical intervention for the injuries, nursing care of these patients includes monitoring airway, assessing for bleeding and infection, and providing patient education on oral care. Depending on the extent of the injury, the patient will likely have their jaw wired. Educate the patient and family on the use of a wire cutter for emergency situations.

obstructive sleep apnea (OSA)

cessation in breathing while sleeping.
Must occur a minimum of 5x/hour (can be hundreds/night)
lasts from 10sec – greater than 1 minute with each episode

Risk factors of obstructive sleep apnea (OSA)

Obesity
Oropharyngeal edema
Family history
Hypothyroidism
short neck with recessed chin
Enlarged tonsils, adenoids, uvula
Cigarette smoking and alcohol or sedative use

Complications of sleep Apnea

HTN
Stroke
Cognitive deficits
Weight gain
Diabetes
Pulmonary disease
Cardiovascular disease
Excessive daytime sleepiness, irritability, inability to concentrate

Diagnostic Tests for sleep apnea

STOP-Bang Sleep Apnea Questionnaire
ABG and TSH level
Sleep study: observation and measurement of the client during sleep.

T/X sleep apnea

Treatment
Lose weight if sleep apnea is caused by obesity
Refrain from alcohol or sedatives
Avoid sleeping on your back (position fixing)
Noninvasive positive-pressure ventilation (NPPV)
BiPap
CPAP
APAP
Modafinil (Attenance,Provigil)

Surgery
Adenoidectomy- to remove excess tissue
Uvulopalatopharyngoplasty (UPPP)- done for those with enlarged tissues- remodels posterior oropharynx
Tracheostomy- bypasses obstruction. Done for those with severe OSA or those who cannot tolerate the CPAP

upper airway obstruction

Airflow blockage in the nose, mouth, pharynx, or larynx

caused by:
trauma
blockages/masses
burns
foreign bodies

Medical emergency!!
Prompt action is required to prevent further patient compromise. Interventions are based on the patient’s presenting signs and symptoms and the cause of the obstruction. If the patient is conscious, perform the Heimlich maneuver. If the patient is unconscious, open the airway by repositioning the head or inserting an oral airway. Suction the patient to remove secretions and perform abdominal thrust. If the obstruction is not able to be cleared, the patient may require an emergency tracheotomy, cricothyroidotomy, or endotracheal intubation.

A 58-year-old woman who has been diagnosed with throat cancer 1 week ago comes to the clinic today to discuss surgical options with her health care provider. She is very tearful and appears sad when the nurse calls her back to the examination room.
Based on her diagnosis, which clinical manifestation will the nurse likely observe in the patient?

A.Hoarseness
B.Severe chest pain
C.Low hemoglobin level (anemia)
D.Numbness and tingling of the face

ANS: A

The patient may experience several different symptoms. The most commonly seen with throat cancer is hoarseness, as well as mouth sores or a lump in the neck. Anemia can result if surgery is performed. Severe pain in the chest can be associated with many different disorders and is not usually linked to throat cancer. Numbness and tingling of the face cannot be observed.
S/Sx of throat cancer: hoarsness, soar throat, difficulty swallowing, mouth sores, ear pain, oral bleeding

When the nurse begins taking the patient’s history, the patient asks, “Did you know that I have throat cancer and may not survive?” What is the appropriate nursing response?

A.”Are you having difficulty swallowing?”
B.”My mother had cancer, so I know how you must be feeling right now.”
C.”I am sure that your cancer can be cured if you follow your doctor’s advice.”
D.”I know you have been diagnosed with cancer. Are you concerned about what the future may hold?”

ANS: D

Although option A is part of an appropriate history, the patient’s need at the moment, represented by her statement, is psychosocial in nature. The nurse should realize that the patient may need psychosocial support. This is the only appropriate therapeutic response. The nurse cannot give her false reassurance (option C), and the nurse should never compare feelings (option B). Head and neck cancer is curable when treated early.

The provider discusses radiation therapy with the patient because her lesion is small and the cure rate is 80% or higher. The patient asks if her voice will return to normal. What is the appropriate nursing response? (Select all that apply.)

A.”At first the hoarseness may become worse.”
B.”The more you use your voice, the quicker it will improve.”
C.”Gargling with saline may help decrease the discomfort in your throat.”
D.”Your voice will improve within 4 to 6 weeks after completion of the therapy.”
E.”You should rest your voice and use alternative communication during the therapy.”

ANS: A, C, D, E

The patient should be taught not to use her voice more than necessary during and after therapy, and to work with family to determine alternative forms of communication until after the radiation therapy. Statements A, C, D, and E are appropriate responses that accurately reflect the normal course of progression after radiation therapy for throat cancer.

After the radiation therapy begins, the patient visits the clinic stating that her throat is sore, she is having difficulty swallowing, and the skin on her throat is red, tender, and peeling.

What patient teaching should the nurse provide?

ANS: For temporary relief of the patient’s sore throat and swallowing difficulty, suggest that she gargle with saline, suck on ice chips, use mouthwash, or use a throat spray with local anesthetics such as lidocaine. For her red, tender, peeling skin, have her avoid exposure to sun, heat, cold, or abrasive treatments such as shaving; wear protective clothing of soft cotton; wash gently with mild soap; and use only lotions or powders prescribed by the radiation oncologist until the area has healed.

The nurse is caring for a patient admitted for treatment of neck and throat cancer. Which intervention should the nurse perform?

A.Encourage hydration with water.
B.Feed the patient if coughing occurs.
C.Encourage the patient to sit in a chair for meals.
D.Encourage the patient to drink juice to address thirst.

ANS: C

Several interventions are necessary to reduce the risk of aspiration. Having the patient sit upright to eat is an important initial step to reduce aspiration. Other interventions include encouraging liquids that are “thick.” Avoiding thin liquids like juice, water, and fruits that produce juice are important strategies to reduce aspiration risks. Coughing may be a sign of difficulty with swallowing or aspiration and requires additional assessment.

What can happen if a pt aspirates? What are they at risk for? What are other ways we can reduce aspiration risk? (speech consult, to chair or HOB all they way up, slow eating)

The nurse is caring for a patient admitted to the ED after experiencing a fall while rock climbing. The patient has several facial fractures. Which objective assessment finding requires immediate intervention?

A.Malaligned nasal bridge
B.Blood draining from one of the nares
C.Crackling of the skin (crepitus) upon palpation
D.Clear glucose positive fluid draining from nares

ANS: D

Blood or clear fluid (cerebrospinal fluid, or CSF) may drain from one or both nares. However, the presence of glucose in the clear drainage indicates that CSF is draining, which could be caused by a skull fracture, a serious complication CSF leak- yellow ring on testing strips. A malaligned nasal bridge and crepitus may be observed when evaluating general facial fractures and would be considered an expected finding.
What would be the priority for this patient? Open airway

The nurse recognizes that a patient with sleep apnea may benefit from which intervention(s)? (Select all that apply.)

A.Weight loss
B.Nasal mask to deliver BiPAP
C.A change in sleeping position
D.Medication to increase daytime sleepiness
E.Position-fixing device that prevents tongue subluxation

ANS: A, B, C, E

All interventions listed are viable interventions that can be of benefit to patients who have sleep apnea. Patients should work with their providers of care to determine the severity of their sleep apnea and which specific interventions would be of most importance to them. Encouraging daytime sleepiness is the opposite of the effect needed for this patient.
What are the signs of sleep apea/who is at risk? Overweight, large neck size, short neckObesity
Oropharyngeal edema
Family history
Hypothyroidism
short neck with recessed chin
Enlarged tonsils, adenoids, uvula
Cigarette smoking and alcohol or sedative use
Complications: HTN
Stroke
Cognitive deficits
Weight gain
Diabetes
Pulmonary disease
Cardiovascular disease
Excessive daytime sleepiness, irritability, inability to concentrate
Treatment:Lose weight if sleep apnea is caused by obesity
Refrain from alcohol or sedatives
Avoid sleeping on your back (position fixing)
Noninvasive positive-pressure ventilation (NPPV)

With which client does the nurse anticipate complications from obstructive sleep apnea following abdominal surgery?

A. 28-year-old who is 80 lbs (36.4 kg) overweight and has a short neck
B. 48-year-old who has type 1 diabetes and chronic sinusitis
C. 58-year-old who has had gastroesophageal reflux disease for 10 years
D. 78-year-old who wears upper and lower dentures and has asthma

•A – overweight and short neck
•Age doesn’t really matter

Mr. Sherwood is a 27-year-old male who had a fractured nose and is recovering from a rhinoplasty. He has a moustache dressing in place that is dry and intact. The nurse observes that the patient is swallowing repeatedly.

What complication does the nurse anticipate? What equipment does the nurse need to assess Mr. Sherwood?

– Posterior nasal bleeding; penlight
•Rationale: Assessing how often the patient swallows after nasal surgery is a priority because repeated swallowing may indicate posterior nasal bleeding. A penlight is used to examine the throat for bleeding.

Mr. Sherwood is concerned because his nose keeps bleeding. He asks the nurse, “Can you tell me again what I can do to keep my nose from bleeding?”

How should the nurse respond to Mr. Sherwood’s question?

Mr. Sherwood is discharged home. The nurse talks with him and his family on how to care for Mr. Sherwood after discharge.

What are some talking points that the nurse should include in discharge teaching for Mr. Sherwood and his family?

•Answer: The nurse may suggest that the patient keep his mouth open while sneezing, not bend over, and avoid coughing and vomiting. Avoid taking aspirin and NSAIDs while the nose heals. Avoid straining during bowel movements.
•Rationale: These activities increase blood pressure causing fragile blood vessels to break and bleed. Teaching the patient to avoid these activities will prevent increase in pressure. Laxatives or stool softeners may help to ease bowel movements. Aspirin and NSAIDs increase bleeding potential.

•The nurse may include instructing the patient to stay in a semi-Fowlers’ position, to move slowly, to keep all follow-up appointments, to call his provider if fever develops, and to use a humidifier.
•Rationale: Providing discharge instructions to Mr. Sherwood and his family allows them to be involved in his care and increases compliance and health care outcomes. Correct positioning and moving slowly decrease chances of bleeding and edema to the area. Following up with the provider allows opportunity for extension of care to complete recovery and identification of problems early.

Asthma occurs in two ways:

•Inflammation
•Airway hyperresponsiveness leading to bronchoconstriction

Asthma is classified based on how well controlled the symptoms are and the patient’s response to the medications to treat the disease process. Status asthmaticus is a severe life-threatening condition that requires prompt intervention

Pathophysiology of asthma

•Intermittent and reversible airflow obstruction affecting airways only, not alveoli

Airway obstruction
•Inflammation
•Airway hyperresponsiveness

causes of asthma

exposure to allergens or irritants; stress, cold, and exercise

Interprofessional Collaborative Care for asthma

Assessment: Noticing
•Physical assessment/clinical manifestations
•Audible wheeze, increased respiratory rate
•Increased cough
•Use of accessory muscles
•”Barrel chest” from air trapping
•Long breathing cycle
•Cyanosis
•Hypoxemia

Symptoms- labored breathing, wheezing, trouble sleeping, frequent cough, feeling tired, feeling short of breath
Common triggers – pollution, dust, smoke, pet dandner, household chemicals, bacteria, viruses, mold

Assessment: Noticing for asthma

Laboratory assessment
•ABGs

Pulmonary function tests
•Forced vital capacity (FVC)
•Forced expiratory volume in first second (FEV1)
•Peak expiratory flow rate (PEFR)

Interventions: Responding to Asthma

1.Self-management education
•Personal asthma action plan
•Control and prevent flair-ups.
•Avoid Triggers
•Use of Peak-flow meter

2.Drug therapy
•Control therapy drugs (used daily)
•Reliever drugs (used to stop an attack)
•Bronchodilators
•Anti-inflammatory agents

Medications for asthma

bronchodilators
•Beta2 agonists
•Short-acting Beta agonist- Albuterol (Proventil, Ventolin)
•Long acting Beta agonist- salmeterol (Serevent)

– Rescue medications are short-acting- Albuterol – carry with them at all times in case of an acute asthma attack – S/E of albuterol- tachycardia, headache dizziness, insomnia, nausea

Cholinergic Antagonists/anticholinergic/long-acting muscarinic antagonists
•Tiotropium (Spiriva), ipratropium (Atrovent)

Anti-Inflammatory Agents
•corticosteroid- fluticasone (Flovent), budesonide (Pulmicort), prednisone
•Leukotriene modifiers- montelukast (Singular), zafirlukast (Accolate)

•Corticosteroids—Disrupt production pathways of inflammatory mediators. The main purpose is to prevent an asthma attack caused by inflammation or allergies (controller drug) – Teach patient to use good mouth care and to check mouth daily for lesions or drainage because these drugs reduce local immunity and increase the risk for local infections, especially Candida albicans (yeast).

•Leukotriene Modifier—Blocks the leukotriene receptor, preventing the inflammatory mediator from stimulating inflammation. The purpose is to prevent an asthma attack triggered by inflammation or allergens.

Asthma treatment continued

Other treatments for Asthma:
•Exercise and activity to promote gas exchange
•Oxygen therapy

Patient Education
•Avoid triggers
•Stop or avoid smoking
•Teach which inhaler is rescue, which is not rescue
•Use rescue inhaler 30 mins before exercise
•Know how to use inhalers (chart 30-7 and 30-8 in book)

status asthmaticus

•Severe, life-threatening, acute episode of airway obstruction
•Intensifies once it begins, often does not respond to common therapy
•Patient can develop pneumothorax and cardiac/respiratory arrest

T/X
IV fluids
potent systemic bronchodilator
steroids
epinephrine
oxygen

chronic obstructive pulmonary disease (COPD)

•Characterized by bronchospasm and dyspnea
•Tissue damage not reversible; increases in severity, eventually leads to respiratory failure
•Cigarette smoking is the greatest risk factor
•Alpha1-antitrypsin deficiency
4th leading cause of morbidity in U.S

The inflammatory response calls cell mediators to the airway. These cell mediators injure cells, over time can permanently damage them.
The repair process after the injury results in scar tissue, non-elastic tissue to form in the lung tissue/airway

2 TYPES:
•Emphysema- alveolar membrane breakdown
•Chronic bronchitis-inflammation and excessive mucus in the bronchiole tubules

Causes: smoking, pollution, genetics

•S/S – chronic cough, mucus, fatigue, SOB, chest pain and discomfort, dyspnea

Emphysema

Pink Puffer- thin and frail-looking

occurs due to high levels of proteases in the lung, which damage the alveoli and cause air trapping in the alveoli.
Emphysema is classified as panlobular, centrilobular, or paraseptal.

– Elastin in the lungs broken down by a high level of proteases damaging alveoli
•Airways collapse/narrow in time
•Flattens/weakens diaphragm
•Hyperinflation of lung and air trapping

– severe dyspnea
-quiet chest
– xray- infiltration with flattened diaphrams.

Chronic Bronchitis

Blue bloater

•Inflammation of bronchi & bronchioles (airway only)
•Inflammation, vasodilation, mucosal edema, congestion, bronchospasm
•Mucus plugs and infection narrow the airway
– overweight and cyanotic
– elevated hemoglobin
– Ronchi and wheezing
– peripheral edema

Complications of COPD

Gas Exchange:
•Hypoxia
•Acidosis
Decreased Oxygen and Increased Carbon Dioxide

•Respiratory Infection
Increased Mucus Production, Inflammation and Bronchospasms

•Heart Failure- Cor pulmonale- Right-sided heart failure caused by pulmonary disease.
•Dysrhythmia
Respiratory Failure

COPD assessment

•Assessment
•Three primary symptoms
•Cough, sputum production, and dyspnea on exertion
•Lung sounds: Wheezes or crackles (rarely)
•Prolonged expiratory phase
•Distant heart sounds
•Orthopneic position, uses chest and abd muscles to help breathe
•Weight loss- due to dyspnea with eating
•Progressive airflow obstruction leads hypoxia
•Clubbing- bulbous enlargement of distal fingers and nails- associated with chronic cyanosis
•Psychosocial

COPD Tests, Education & Nursing interventions

•Laboratory Assessments & Diagnostic Tests:
•Pulmonary function/ spirometry testing
•Chest X-ray- reveals heart and lung size
•CT of the Chest- shows hyperinflation and/or bullae
•ECG- dysrhythmias
•ABG- blood gases

Education
•Quitting smoking is the most effective way to slow the progression of COPD
•Energy conservation
•Small frequent meals (high calorie/protein)- stay hydrated!
•Rest before meals if dyspneic

Interventions
Other nursing care strategies include performing chest physiotherapy to facilitate the clearance of secretions, encouraging the patient to cough and take deep breathes as well as the use of the incentive spirometer to facilitate airway clearance. Patients may also benefit from noninvasive ventilation to assist with ventilation and oxygenation.
•Patients with emphysema often have limited levels of activity due to their disorder; it is important to implement strategies to prevent deep vein thrombosis such as anti-embolism stockings or sequential compression stockings. Patients who require surgical management will require nursing interventions, including preparing the patient for the procedure and post-operative management of the patient.

COPD treatment and meds

COPD Exacerbation treatment
•Low flow oxygen- DO NOT LIMIT 1-2 L, not above 4L
•Rest (limit activities to focus on breathing and oxygenation)
•Increase fluids (to help thin secretions making it easier to expel them)
•Continuous pulse oximetry monitoring

•Meds- Nebulizer treatments with normal saline or a mucolytic agent such as acetylcysteine (Mucosil, Mucomys & same as asthma
•And possible anti-tussives like guaifenesin and dextromethorphan (Mucinex DM) also raises the cough threshold.
Sat goal is 88-92% they may not get higher than that!

cystic fibrosis

•An inherited, recessive, chronic, progressive, and frequently fatal disease of the body’s exocrine mucus-producing glands that primarily affects the respiratory, digestive, and intestinal systems and pancreas.

Each parent must pass the recessive gene for the child to get CF.

Leads to lung infections, poor digestion, poor food absorption, and male sterility

•Genetic disease affecting many organs, lethally impairing pulmonary function
•Blocked chloride transport, producing thick mucus with low water content
•Mucus plugs up glands, causing atrophy and organ dysfunction

Signs and symptoms of Cystic fibrosis

Non-pulmonary symptoms
•Abdominal distention
•GERD, rectal prolapse, foul-smelling stools, steatorrhea
•Malnourishment, vitamin deficiencies

Pulmonary symptoms
•Respiratory infections
•Chest congestion and sputum production
•Decreased pulmonary function
•Limited exercise tolerance

Nonsurgical Management of Cystic Fibrosis

•Nutritional management
•Preventive/maintenance therapy
•Exacerbation therapy

Nutrition
•Teach the importance of pancreatic enzyme replacement, adequate oral fluid intake, and diet to promote removal of secretions
•Monitor stools for bulky, foul-smelling stool (indicates malabsorption)
•Give supplemental iron
•Monitor daily weight
•Daily chest CPT

Implementation of daily chest physiotherapy with postural drainage will assist in drainage of secretions and improve oxygenation. Infection is common in patients with this disorder; early identification and treatment are vital. Patients may also benefit from noninvasive positive-pressure ventilation to improve ventilation. Other nursing interventions include pre-operative and post-operative care for patients undergoing a lung transplant

Surgical Management CF

•Lung transplantation
•Does not cure
•Extends life by 1 to 15 years
•Transplant rejection rate is high
•Patient at continued risk for lethal pulmonary infections

Pulmonary Arterial Hypertension (PAH)
idiopathic pulmonary hypertension

•Occurs in absence of other lung disorders; cause unknown
•Blood vessel constriction with increasing vascular resistance in the lung
•Heart fails (cor pulmonale)
•Without treatment, death within 2 years

•Nursing care of the patient with pulmonary hypertension is dependent on the severity classification. Nursing care interventions include administration of prescribed medications (such as calcium channel blockers, endothelin-receptor agonist, natural and synthetic prostacyclin agents, and guanylate cyclase stimulators), administration of oxygen as prescribed, assisting with placement of a pulmonary artery catheter to monitor pressures, and preparation of the patient for surgical intervention.

Idiopathic pulmonary fibrosis

•Common, restrictive lung disease
•Progressive disease, with few periods of remission
•Loss of cellular regulation

Lung injury> Inflammation >Fibrosis> Scarring

• Corticosteroids, other immunosuppressants mainstays of therapy


•The patient usually is an older adult with a history of cigarette smoking, chronic exposure to inhalation irritants, or exposure to the drugs amiodarone (Cordarone) or ambrisentan (Letairis, Volibris). Most patients have progressive disease with few remission periods. Even with proper treatment, most patients usually survive less than 5 years after diagnosis
Causes of Pulmonary Fibrosis – The cause of IPF is unknown and the course of the disease is not predictable

•Pulmonary Fibrosis
•Nursing interventions for a patient with pulmonary fibrosis include administration of the medications and oxygen as prescribed and preparation of the patient for a lung transplant. Patient and family education include identification of community resources, monitoring for respiratory infections and identification of when to seek evaluation, and oxygen safety. Other strategies for patients and families facing end stages of the disease include providing information to the patient and family regarding hospice care.

Lung cancer

Lung cancer develops due to a loss of cellular regulation; the most common site of lung cancer is the epithelium of the bronchial tree. Lung cancers are classified as small cell lung cancer (SCLS) and non-small cell lung cancer (NSCLC).
Lung cancers impede gas exchange, which disrupts the acid-base balance. If left untreated, lung cancer can metastasize to other organs and tissues. Early recognition and aggressive treatment are essential to improve the patient’s prognosis. Primary prevention is the key to reducing the incidence of lung cancer.

Lung Cancer Warning Signs

•Hoarseness
• Change in respiratory pattern
• Persistent cough or change in cough
• Blood-streaked sputum
• Rust-colored or purulent sputum
• Frank hemoptysis
• Chest pain or chest pressure
• Shoulder, arm, or chest wall pain
• Recurring episodes of pleural effusion, pneumonia, or bronchitis
• Dyspnea
• Fever associated with one or two other signs
• Wheezing
• Weight loss
• Clubbing of the fingers

Nonsurgical Management lung cancer

•Chemotherapy treatment of choice for lung cancers, especially small cell lung cancer (SCLC) Side effects that occur with chemotherapy for lung cancer include chemotherapy-induced nausea and vomiting (CINV), alopecia (hair loss), open sores on mucous membranes (mucositis), immunosuppression with neutropenia, anemia, thrombocytopenia (decreased numbers of platelets), and peripheral neuropathy.

•Targeted therapy- •common in the treatment of non-small cell lung cancer (NSCLC). These agents take advantage of one or more differences in cancer cell growth or metabolism that is either not present or only slightly present in normal cells. Agents used as targeted therapies work to disrupt cancer cell division

•Radiation therapy may be performed before surgery to shrink the tumor and make resection easier

•Photodynamic therapy may be used to remove small bronchial tumors using targeted laser lights

Surgical Management lung cancer

•Lobectomy
•Pneumonectomy
•Segmentectomy
•Wedge resection

•Post Op- most pt have very high levels of intense pain afer lung surgery – may have PCA
•Assess 02 sats continuously and perform lung assessment frequently- usually have suppl O2 via nasal cannula
•Coughing- splint any coughing with a pillow
•Use incentive spirometer q2 hour and get up to edge of bed or chair as soon as able

chest tube placement

•To remove fluid or air from the pleural space.
•Usually inserted into 5th intercostal space creating a communication between the outside atmosphere and the pleural space. . . . Allows air/fluid to move out.
•As the air/fluid is removed, the pleural space is decompressed to restore negative pressure.
•The chest tube is hooked to a canister measuring fluid output, it may also be hooked up to suction.
•The canister has a water seal, which prevents air from flowing back into the pleural space.

Chest Tube Drainage System

nursing management of a chest tube
you are going to check them for patency, make sure they are draining, make sure there is still pressure going to the chest tube so that the negative pressure environment in the lung can be maintained- you want to note the amount of drainage per hour (usually you make a small mark with perm marker on the drain) color/clarity of the drainiage- usually it is sereous to serosang but occasionally it is white (A chylothorax (ky-low-thor-ax) is the buildup of chyle (kile), a milky white fluid, in the space around the lungs. … Fluid can collect between the pleura layers (called the pleural space). A chylothorax happens when the lymphatic system starts leaking chyle fluid into the pleural space.)


This fluid is measured hourly during the first 24 hours. The fluid in chamber one must never fill to the point that it comes into contact with any tubes! If the tubing from the patient enters the fluid, drainage stops and can lead to a tension pneumothorax.

A chest tube falling out is an emergency. Immediately apply pressure to chest tube insertion site and apply sterile gauze or place a sterile vasaline/occlusive dressing gauze and dry dressing over insertion site and ensure tight seal. Apply dressing when patient exhales. If patient goes into respiratory distress, call a code

Nursing Care for Chest Tubes-The Patient

•Ensure that the dressing on the chest around the tube is tight and intact. Depending on agency policy and the surgeon’s preference, reinforce or change loose dressings.
•Assess for difficulty breathing.
•Assess breathing effectiveness by pulse oximetry.
•Listen to breath sounds for each lung.
•Check alignment of trachea.
•Check tube insertion site for condition of the skin. Palpate area for puffiness or crackling that may indicate subcutaneous emphysema.
•Observe site for signs of infection (redness, purulent drainage) or excessive bleeding.
•Check to see if tube “eyelets” are visible.
•Assess for pain and its location and intensity and administer drugs for pain as prescribed.
•Assist patient to deep breathe, cough, perform maximal sustained inhalations, and use incentive spirometry.
•Reposition the patient who reports a “burning” pain in the chest.

Nursing Care for Chest Tubes-The Drainage System

Keep drainage system lower than the level of the patient’s chest.

•Do not “strip” the chest tube.

Keep the chest tube as straight as possible from the bed to the suction unit, avoiding kinks and dependent loops. Extra tubing can be loosely coiled on the bed.

• Ensure that the chest tube is securely taped to the connector and that the connector is taped to the tubing going into the collection chamber.

• Assess bubbling in the water-seal chamber; should be gentle bubbling on patient’s exhalation, forceful cough, position changes.

• Assess for “tidaling” (rise and fall of water in chamber three with breathing).

• Check water level in the water-seal chamber and keep at the level recommended by the manufacturer.
• Check water level in the suction control chamber and keep at the level prescribed by the surgeon (unless dry suction system is used).

Clamp the chest tube only for brief periods to change the drainage system or when checking for air leaks.

• Check and document amount, color, and characteristics of fluid in the collection chamber as often as needed according to the patient’s condition and agency policy.

• Empty collection chamber or change the system before the drainage makes contact with the bottom of the tube.

• When a sample of drainage is needed for culture or
other laboratory test, obtain it from the chest tube; after cleaning chest tube, use a 20-gauge (or smaller) needle and draw up specimen into a syringe.

Nursing Care for Chest Tubes-Emergencies

•Tracheal deviation
•Sudden onset or increased intensity of dyspnea
•Oxygen saturation less than 90%
•Drainage greater than 70 mL/hr
•Visible eyelets on chest tube
•Chest tube falls out of the patient’s chest (first, cover the area with dry, sterile gauze)
•Chest tube disconnects from the drainage system (first, put end of tube in a container of sterile water and keep below the level of the patient’s chest)
•Drainage in tube stops (in the first 24 hours)

Interventions for Palliation

•Oxygen therapy
•Drug therapy
•Radiation therapy
•Thoracentesis and pleurodesis
•Dyspnea management
•Pain management
•Hospice care

Based on the patient’s diagnosis, which clinical manifestations would the nurse expect to see when assessing this patient? (Select all that apply.)

A.Bradycardia
B.Shortness of breath
C.Use of accessory muscles
D.Sitting in a forward posture
E.Barrel chest appearance

ANS: B, C, D, E

The patient with COPD often has a barrel chest appearance, is short of breath, and may use accessory muscles when breathing. These patients tend to move slowly and are slightly stooped. Usually they sit with a forward-bending posture. With severe dyspnea, they exhibit activity intolerance and activities such as bathing and grooming are avoided.

While the Rapid Response Team is at the bedside, the patient’s healthcare provider arrives. The provider writes several orders.

Which order is most important for the nurse to implement immediately?

A.Transfer to ICU
B.Increase O2 to 3 L per nasal cannula
C.ABGs 30 minutes after oxygen is increased
D.Methylprednisolone sodium succinate (Solu-Medrol) 40 mg IVP

ANS: B

All of the provider’s orders are very important, but based on the patient’s severe shortness of breath, the first thing that should be done is to increase her oxygen. Once her oxygen is increased, the nurse should note the time and remember to call for stat ABGs in 30 minutes. The patient should then be transferred to the ICU as soon as possible. Once the patient arrives in the ICU, they can administer the one-time dose of Solu-Medrol.

A patient with COPD presents for a routine follow up. The patient smokes 1 PPD. Which statement by the patient causes the nurse to suspect an increase in dyspnea?

A.”I bought a new pillow so I could prop myself up at night to sleep.”
B.”I have a productive cough in the morning.”
C.”I have gained weight since I was here last.”
D.”The patient is well groomed and is sitting in a tripod position.”

ANS: A

Patients with COPD, who smoke, may have a productive morning cough. Weight loss often occurs when dyspnea is increased due to the increased metabolic demand. A tripod or orthopneic position is common with COPD and when combined with a disheveled appearance may indicate an increase in dyspnea. Buying a new pillow indicates that the patient must sleep propped up because breathing is worse while lying down. They may not recognize the increased dyspnea and they try to compensate by using multiple pillows in order to rest.

The nurse is assessing a patient with a chest tube following a pneumonectomy. Which assessment finding requires intervention?

A.Bandage around the posterior tube is loose.
B.2 cm of water is in the second chest tube chamber.
C.The water in the water seal chamber rises and falls with inhalation/exhalation.
D.Bubbling present in the water seal chamber when the patient coughs.

ANS: A

After lung surgery, two tubes, anterior and posterior, are used. Dressings around the wound should not be loose. The wounds should be covered with airtight dressings.

A patient with a history of asthma is having shortness of breath. The nurse discovers that the peak flowmeter indicates a peak expiratory flow (PEF) reading that is in the red zone. What is the priority nursing action?

A.Administer the rescue drugs.
B.Take the patient’s vital signs.
C.Notify the patient’s prescriber.
D.Repeat the PEF reading to verify the results.

ANS: A

A PEF reading in the red zone indicates a range that is 50% below the patient’s personal best PEF reading and indicates serious respiratory obstruction. The patient needs to receive rescue drugs immediately, and then the prescriber should be notified. Repeating the PEF reading and taking vital signs are also important, but doing so first delays the administration of the rescue drugs.

Let’s talk about meds! Ok so when someone has an asthma attack what is first? Albeuterol rescue inhaler (SABA short acting bronchodilator), THEN what would be give? Then we could give them their inhaled steroid (pulmicort or fluticasone) OR if we are in the hospital they sometimes can get an IV corticosteroid like methyprednisalone as well to reduce inflacmation

Seasonal Influenza

•Highly contagious acute viral respiratory infection: Strains A, B and C
•Flu season ranges from October – May with peak in February
•Severe headache, muscle ache, fever, chills, fatigue, weakness, anorexia
•Patient is contagious 24hours before to up to 5 days after onset of symptoms.
•Vaccination is advisable
•Age, Chronic illness, Pregnancy, Immunocompromised at high risk
•Antiviral agents may be effective if started within 24 to 48 hours of symptoms

Pandemic Influenza

•Mostly prevalent among animals and birds; virus can mutate, becoming infectious to humans
These symptoms can progress to pneumonia. Preventative measures, including yearly influenza vaccination, can prevent pandemic issues.
•Examples
•H1N1 (swine flu)
•H5N1 (bird flu)
•Strict isolation precautions
•Antiviral drugs
•Oseltamivir (Tamiflu), zanamivir (Relenza)

Pneumonia

•Acute or Chronic infection of one or both lungs caused by bacteria, virus, chemical irritant, fungus, mycoplasms, toxic gasses, aspiration of water, food, fluid (saliva), vomit…
•Increased risk population (table 31-1)
•Less than 2 yrs
•Older than 65 yrs
•Immunocompromised patient
•Pneumocystis carinii pneumonia (PCP) is a type of fungal infection that mostly affects pts with HIV. Can be deadly as this affects immunocompromised patients.
•Types of Pneumonia
•Community Acquired pneumonia (CAP)-
•Hospital Acquired pneumonia (HAP)
•Ventilator-associated pneumonia (VAP)
•Health Care-associated

Pneumonia Assessment

•Assessment
•Fever, chills/rigors, sweats
•Cough with or without sputum
•Pleuritic chest pain/weakness
•Dyspnea
•Malaise
•Fatigue
•Headaches
•Anorexia
•RR>20, O2 may be needed, HR increased
•Lung sounds: crackles auscultated, wheezing

•Older adults- hypotensive with orthostatic hypotension d/t vasodilation and dehydration – potentially confusion/falls

Pneumonia tests, interventions and treatment

Diagnostic Tests
•Chest x-ray
•CBC
•Sputum sample
•Blood urea nitrogen (dehydration)

Interventions
•Oxygen therapy
•Incentive spirometry
•Fluids

Medication
•Antibiotic therapy
•Pneumonia vaccine
Bronchodilators, especially beta2 agonists (see Chart 30-6), are prescribed when bronchospasm is present. They can be given by nebulizer or metered-dose inhaler. Inhaled or IV steroids are used with acute pneumonia when airway swelling is present. Expectorants such as guaifenesin (Mucinex) may be used. Antibiotics are used if bacterial infection is suspected culprit

Appropriate antibiotics are prescribed. A chest tube(s) to closed-chest drainage is used to promote lung expansion and drainage

Pneumonia education

•Frequent oral hygiene
•Good hand washing to reduce spread of infection
•If pt is ventilated, needs oral hygiene every 1-2hrs with chlorhexidine
•Change oxygen tubing with contamination
•Importance of vaccination-Pneumovax or Prevnar 13
•Pt receiving antibiotics that is having rigors, increased fever, or increased cough with sputum may indicate the medication is ineffective.
•Increase oral fluids to 2L/day to help thin sputum and clear secretions
•Incentive Spirometer every 1h WA, or TCDB

•The Joint Commission National Patient Safety Goals [NPSGs] recommend that nurses especially encourage adults older than 65 years and those with a chronic health problem to receive immunization against pneumonia. There are two pneumonia vaccines: pneumococcal polysaccharide vaccine (PPSV 23), known as Pneumovax; and pneumococcal conjugate vaccine (PCV-13), known as Prevnar 13

Pulmonary empyema

•is a collection of pus in the pleural space most commonly caused by pulmonary infection. When empyema is present, GAS EXCHANGE can be impaired by both reduced lung diffusion and reduced effective ventilation. Empyema fluid is thick, opaque, exudative, and foul smelling.

Tuberculosis

Airborne precautions

•Chronic infection of the lung caused by mycobacterium results in tubercles

•Tubercles- nodules or swelling of lymphocytes and epithelioid cells that form lesions in lung tissue

Assessment: (symptoms are often vague)
•2+ weeks of cough, fever or weight loss
•Weight loss over 3lbs/wk is considered significant
•Night sweats, weakness, and chills may be present with hemoptysis when the infection has progressed

Diagnostic Tests
•PPD or Mantoux skin testing. Results are read 48-72h after injection
•Chest Xray
•Sputum culture
•NAAT

Tuberculosis treatment and education

Medication
•Rifampin (RIF)
•Isoniazid (INH)
•Pyrazinamide (PZA)
•Ethambutol (EMB)
•These medications are given in different combinations that are tailored to the patient’s individual infection
•All cause hepatotoxicity!- check liver enzymes

Nursing Practice
•Importance of medication regimen adherence
•Direct observation Therapy- PHN observes patients in their homes with their medication regimens
•Respiratory isolation
•Education
•How to minimize the spread of TB
•Potential of resistant TB if medication not followed
•Encourage to eat a healthy diet (anorexia is common)

Rhinosinusitis

•Inflammation of nasal mucosa
•”Hay fever” or “allergies”

Manifestations
•Headache
•Nasal irritation and congestion
•Sneezing and rhinorrhea

Interventions
•Antihistamines, leukotriene inhibitors, mast cell stabilizers, decongestants, antipyretics, antibiotics
•Supportive therapy
•Complementary and alternative therapy—vitamin C, zinc

Peritonsillar Abscess (PTA)

•Complication of acute tonsillitis
•Manifestations
•Pus causing one-sided swelling with deviation of the uvula
•Trismus and difficulty breathing
•Bad breath, swollen lymph nodes
•Treatment
•Percutaneous needle aspiration of abscess
•Antibiotics

•. Diagnosis is usually made based on the patient’s symptoms, but needle aspiration and culture of pus collected is the preferred test.

•trouble swallowing, trouble breathing, difficulty speaking, drooling, or any other signs of potential airway obstruction would be ER visit

Pertussis

•Highly contagious, bacterial respiratory infection.
•First phase (catarrhal)
•Symptoms resemble the common cold
•Second phase (paroxysmal)
•Severe coughing, coughing spasms
•Thick exudate in the small airways
•Third phase (convalescent)
•Recovery (can last for months)

•During the catarrhal phase, which lasts for one to two weeks, the patient appears to have a common cold. The nose and airways become filled with mucus. During the paroxysmal phase, the patient experiences severe coughing episodes that may result in vomiting. During the convalescent phase, the patient is recovering; this stage can last several months.

Coccidioidomycosis

•Coccidioidomycosis is a fungal infection caused by the Coccidioides organism
•Common in the desert southwest regions of the United States, Mexico, and Central and South America
•Also known as “Valley Fever”
•Respiratory symptoms, headache, muscle aches, chest pain, fever
•Treated with antifungal medication

•The incubation period is one to four weeks. Inhaled spores stimulate an inflammatory process in the lungs resulting in an infection. These spores are present in the soil and, when disturbed, are released into the air.
can lead to development of an actual pulmonary infection within 1 to 4 weeks after exposure

risk factor
•Agricultural workers

resemble other respiratory infections with fever, cough, headache, muscle aches, chest pain, and night sweats. Bone and joint pain indicates more severe infection. Often the disorder is misdiagnosed and mistreated as influenza or pneumonia. Neither antibacterial drugs nor antiviral drugs are effective therapy. The disease can become widespread and cause symptoms of hemoptysis, meningitis, and involvement of the skin, adrenal glands, liver, and spleen. It also can become chronic and debilitating

The nurse understands that which of the following is the most common symptom of pneumonia in the older adult patient?

A.Fever
B.Cough
C.Confusion
D.Weakness

ANS: C

The older adult with pneumonia often has weakness, fatigue, lethargy, confusion, and poor appetite. Fever and cough may be absent, but hypoxemia is usually present. The most common manifestation of pneumonia in the older adult patient is confusion from hypoxia rather than fever or cough.

What do we need to teach our older adults about pneumonia:
Complete antibiotics as prescribed, rest , drink fluids, and minimize contact with crowds

A patient presents to the primary healthcare provider’s office with fever, ear pressure, sore throat, nasal congestion, and poor response to decongestants. What condition does the nurse suspect?

A.Pneumonia
B.Peritonsillar abscess
C.Tuberculosis exposure
D.Bacterial rhinosinusitis

ANS: D

The presented symptoms indicate bacterial rhinosinutsitis. Symptoms in bacterial infection include purulent nasal drainage with postnasal drip, sore throat, fever, erythema, swelling, fatigue, dental pain, and ear pressure. A lack of response to decongestants can also be indicative of a bacterial infection.

A nursing student is teaching a 72-year-old patient about the importance of the pneumonia vaccination. Which teaching requires intervention by the nurse? (Select all that apply.)

A.”You will only need one vaccine called Pneumovax.”
B.”You will need two vaccines to prevent pneumonia.”
C.”If you have had the Prevnar vaccine, then you will not need the Pneumovax vaccine.”
D.”Since you are over 64 years old, only the flu vaccine is suggested.”
E.”You will receive the Prevnar vaccine about a year after the Pneumovax vaccine.”

ANS: A, C, D,

There are two pneumonia vaccines: pneumococcal polysaccharide vaccine (PPSV 23), known as Pneumovax; and pneumococcal conjugate vaccine (PCV-13), known as Prevnar 13. The CDC recommends that adults older than 65 years be vaccinated with both, first with Prevnar 13 followed by Pneumovax about 6 to 12 months later. Adults who have already received the Pneumovax should have Prevnar 13 about a year or more later. These recommendations also apply to adults between 19 and 64 years of age who have specific risk factors such as chronic illnesses (CDC, 2015j).

Leave a Comment

Scroll to Top